You are on page 1of 105

APPLIED PATHOPHYSIOLOGY A CONCEPTUAL APPROACH TO THE MECHANISMS OF

DISEASE 3RD EDITION

APPLIED PATHOPHYSIOLOGY A CONCEPTUAL


APPROACHTO THE MECHANISMS OFDISEASE 3RD
EDITION BRAUN TEST BANK QUESTIONS AND
ANSWER KEY AFTER EVERY CHAPTER 100%
GUARANTEED PASS|ALL CHAPTERS INCLUDED

Applied Pathophysiology A Conceptual Approach to theMechanisms


of Disease 3rd Edition Braun Test Bank

Chapter 1Introduction to Pathophysiology


APPLIED PATHOPHYSIOLOGY A CONCEPTUAL APPROACH TO THE MECHANISMS OF
DISEASE 3RD EDITION

1. The nucleus , which is essential for function and survival of the cell.
A) is the site of protein synthesis
B) contains the genetic code
C) transforms cellular energy
D) initiates aerobic metabolism
2. Although energy is not made in mitochondria, they are known as the power
plants of the cell because they:
A) contain RNA for protein synthesis.
B) utilize glycolysis for oxidative energy.
C) extract energy from organic compounds.
D) store calcium bonds for muscle contractions.
3. Although the basic structure of the cell plasma membrane is formed by a lipid
bilayer, most of the specific membrane functions are carried out by: A) bound
and transmembrane proteins.

B) complex, long carbohydrate chains.


C) surface antigens and hormone receptors.
D) a gating system of selective ion channels.
4. To effectively relay signals, cell-to-cell communication utilizes chemical
messenger systems that:
A) displace surface receptor proteins.
B) accumulate within cell gap junctions.
C) bind to contractile microfilaments.
D) release secretions into extracellular fluid.
APPLIED PATHOPHYSIOLOGY A CONCEPTUAL APPROACH TO THE MECHANISMS OF
DISEASE 3RD EDITION

5. Aerobic metabolism, also known as oxidative metabolism, provides energy by:


A) removing the phosphate bonds from ATP.
B) combining hydrogen and oxygen to form water.
C) activating pyruvate stored in the cytoplasm.
D) breaking down glucose to form lactic acid.
6. Exocytosis, the reverse of endocytosis, is important in into the
extracellular fluid.
A) Engulfing and ingesting fluid and proteins for transport
B) Killing, degrading, and dissolving harmful microorganisms
C) Removing cellular debris and releasing synthesized substances
D) Destruction of particles by lysosomal enzymes for secretion
7. The process responsible for generating and conducting membrane potentials is:
A) diffusion of current-carrying ions.

B) millivoltage of electrical potential.


C) polarization of charged particles.
D) ion channel neurotransmission.
8. Epithelial tissues are classified according to the shape of the cells and the
number of layers. Which of the following is a correctly matched description and
type of epithelial tissue?
A) Simple epithelium: cells in contact with intercellular matrix; some do not
extend to surface
B) Stratified epithelium: single layer of cells; all cells rest on basement
membrane
C) Glandular epithelium: arise from surface epithelia and underlying
connective tissue
D) Pseudostratified epithelium: multiple layers of cells; deepest layer rests on
basement membrane
9. Connective tissue contains fibroblasts that are responsible for: A)
providing a fibrous framework for capillaries.
APPLIED PATHOPHYSIOLOGY A CONCEPTUAL APPROACH TO THE MECHANISMS OF
DISEASE 3RD EDITION

B) synthesis of collagen, elastin, and reticular fibers.


C) forming tendons and the fascia that covers muscles.
D) filling spaces between tissues to keep organs in place.
10. Although all muscle tissue cells have some similarities, smooth muscle (also
known as involuntary muscle) differs by:
A) having dense bodies attached to actin filaments.
B) containing sarcomeres between Z lines and M bands.
C) having rapid contractions and abundant cross-striations.
D) contracting in response to increased intracellular calcium.
11. Which of the following aspects of the function of the nucleus is performed by
ribosomal RNA (rRNA)?
A) Copying and carrying DNA instructions for protein synthesis
B) Carrying amino acids to the site of protein synthesis
C) Providing the site where protein synthesis occurs
D) Regulating and controlling protein synthesis
12. Breakdown and removal of foreign substances and worn-out cell parts are
performed by which of the following organelles?
A) Lysosomes
B) Golgi apparatus
C) Ribosomes
D) Endoplasmic reticulum (ER)
13. Impairment in the function of peroxisomes would result in: A)
inadequate sites for protein synthesis.

B) an inability to transport cellular products across the cell membrane.


C) insufficient energy production within a cell.
D) accumulation of free radicals in the cytoplasm.
14. After several months of trying to conceive, a couple is undergoing fertility
testing. Semen analysis indicates that the mans sperm have decreased motility, a
finding that is thought to underlie the couples inability to become pregnant.
APPLIED PATHOPHYSIOLOGY A CONCEPTUAL APPROACH TO THE MECHANISMS OF
DISEASE 3RD EDITION

Which of the following cellular components may be defective within the mans
sperm?
A) Ribosomes
B) Microtubules
C) Mitochondria
D) Microfilaments
15. Which of the following statements is true of glycolysis? A)
Glycolysis requires oxygen.

B) Glycolysis occurs in cells without mitochondria.


C) Glycolysis provides the majority of the bodys energy needs.
D) Glycolysis produces energy, water, and carbon dioxide.
16. Which of the following membrane transport mechanisms requires the greatest
amount of energy?
A) Facilitated diffusion
B) Passive transport
C) Vesicular transport
D) Simple diffusion
17. A male patient with a diagnosis of type 1 diabetes mellitus is experiencing
hyperglycemia because he lacks sufficient insulin to increase the availability of
glucose transporters in his cell membranes. Consequently, his cells lack
intracellular glucose and it accumulates in his blood. Which of the following
processes would best allow glucose to cross his cell membranes?
A) Facilitated diffusion
B) Simple diffusion
C) Secondary active transport
D) Endocytosis
18. Which of the following statements is true of skeletal muscle cells?
A) Skeletal muscle cells each have an apical, lateral, and basal surface.
B) They are closely apposed and are joined by cell-to-cell adhesion molecules.
APPLIED PATHOPHYSIOLOGY A CONCEPTUAL APPROACH TO THE MECHANISMS OF
DISEASE 3RD EDITION

C) Their basal surface is attached to a basement membrane.


D) Skeletal muscle is multinucleated, lacking true cell boundaries.
19. Which of the following body tissues exhibits the highest rate of turnover and
renewal?
A) The squamous epithelial cells of the skin
B) The connective tissue supporting blood vessels
C) The skeletal muscle that facilitates movement
D) The nervous tissue that constitutes the central nervous system
20. A patient with a pathophysiologic condition that affects the desmosomes is most
likely to exhibit:
A) impaired contraction of skeletal and smooth muscle.
B) weakness of the collagen and elastin fibers in the extracellular space.
C) impaired communication between neurons and effector organs.
D) separation at the junctions between epithelial cells.

Answer Key

1. B
2. C
3. A
4. D
5. B
6. C
7. A
8. C
9. B
10. A
11. C
12. A
APPLIED PATHOPHYSIOLOGY A CONCEPTUAL APPROACH TO THE MECHANISMS OF
DISEASE 3RD EDITION

13. D
14. B
15. B
16. C
17. A
18. D
19. A
20. D

Chapter 2 Altered Cells and Tissues

1. Ischemia and other toxic injuries increase the accumulation of intracellular


calcium as a result of:
A) release of stored calcium from the mitochondria.
B) improved intracellular volume regulation.
C) decreased influx across the cell membrane.
D) attraction of calcium to fatty infiltrates.
2. The patient is found to have liver disease, resulting in the removal of a lobe of his
liver. Adaptation to the reduced size of the liver leads to of the
remaining liver cells.
A) metaplasia
B) organ atrophy
C) compensatory hyperplasia
D) physiologic hypertrophy
3. A person eating peanuts starts choking and collapses. His airway obstruction is
partially cleared, but he remains hypoxic until he reaches the hospital. The
prolonged cell hypoxia caused a cerebral infarction and resulting in
the brain.
A) caspase activation
B) coagulation necrosis
C) rapid phagocytosis
APPLIED PATHOPHYSIOLOGY A CONCEPTUAL APPROACH TO THE MECHANISMS OF
DISEASE 3RD EDITION

D) protein p53 deficiency


4. Bacteria and viruses cause cell damage by , which is unique from the
intracellular damage caused by other injurious agents.
A) disrupting the sodium/potassium ATPase pump
B) interrupting oxidative metabolism processes
C) replicating and producing continued injury
D) decreasing protein synthesis and function
5. The patient has a prolonged interruption in arterial blood flow to his left kidney,
causing hypoxic cell injury and the release of free radicals. Free radicals damage
cells by:
A) destroying phospholipids in the cell membrane.
B) altering the immune response of the cell.
C) disrupting calcium storage in the cell.
D) inactivation of enzymes and mitochondria.
6. Injured cells have impaired flow of substances through the cell membrane as a
result of:
A) increased fat load.
B) altered permeability.
C) altered glucose utilization.
D) increased surface receptors.
7. Reversible adaptive intracellular responses are initiated by:
A) stimulus overload.
B) genetic mutations.
C) chemical messengers.
D) mitochondrial DNA.
8. Injured cells become very swollen as a result of: A)
increased cell protein synthesis.

B) altered cell volume regulation.


APPLIED PATHOPHYSIOLOGY A CONCEPTUAL APPROACH TO THE MECHANISMS OF
DISEASE 3RD EDITION

C) passive entry of potassium into the cell.


D) bleb formation in the plasma membrane.
9. A diabetic patient has impaired sensation, circulation, and oxygenation of his
feet. He steps on a piece of glass, the wound does not heal, and the area tissue
becomes necrotic. The necrotic cell death is characterized by:
A) rapid apoptosis.
B) cellular rupture.
C) shrinkage and collapse.
D) chronic inflammation.
10. A 99-year-old woman has experienced the decline of cell function associated
with age. A group of theories of cellular aging focus on programmed: A)
changes with genetic influences.

B) elimination of cell receptor sites.


C) insufficient telomerase enzyme.
D) DNA mutation or faulty repair.
11. An 89-year-old female patient has experienced significant decreases in her
mobility and stamina during a 3-week hospital stay for the treatment of a
femoral head fracture. Which of the following phenomena most likely accounts
for the patients decrease in muscle function that underlies her reduced mobility?
A) Impaired muscle cell metabolism resulting from metaplasia
B) Dysplasia as a consequence of inflammation during bone remodeling
C) Disuse atrophy of muscle cells during a prolonged period of immobility
D) Ischemic atrophy resulting from vascular changes while on bedrest
12. A 20-year-old college student has presented to her campus medical clinic for a
scheduled Papanicolaou (Pap) smear. The clinician who will interpret the smear
will examine cell samples for evidence of:
A) changes in cell shape, size, and organization.
B) the presence of unexpected cell types.
C) ischemic changes in cell samples.
D) abnormally high numbers of cells in a specified field.
APPLIED PATHOPHYSIOLOGY A CONCEPTUAL APPROACH TO THE MECHANISMS OF
DISEASE 3RD EDITION

13. Which of the following pathophysiologic processes is most likely to result in


metastatic calcification?
A) Benign prostatic hyperplasia
B) Liver cirrhosis
C) Impaired glycogen metabolism
D) Hyperparathyroidism
14. Despite the low levels of radiation used in contemporary radiologic imaging, a
radiology technician is aware of the need to minimize her exposure to ionizing
radiation. What is the primary rationale for the technicians precautions? A)
Radiation stimulates pathologic cell hypertrophy and hyperplasia.
B) Radiation results in the accumulation of endogenous waste products in the
cytoplasm.
C) Radiation interferes with DNA synthesis and mitosis.
D) Radiation decreases the action potential of rapidly dividing cells.
15. The parents of a 4-year-old girl have sought care because their daughter has
admitted to chewing and swallowing imported toy figurines that have been
determined to be made of lead. Which of the following blood tests should the
care team prioritize?
A) White blood cell levels with differential
B) Red blood cell levels and morphology
C) Urea and creatinine levels
D) Liver function panel
16. A 70-year-old male patient has been admitted to a hospital for the treatment of a
recent hemorrhagic stroke that has left him with numerous motor and sensory
deficits. These deficits are most likely the result of which of the following
mechanisms of cell injury?
A) Free radical injury
B) Hypoxia and ATP depletion
C) Interference with DNA synthesis
D) Impaired calcium homeostasis
APPLIED PATHOPHYSIOLOGY A CONCEPTUAL APPROACH TO THE MECHANISMS OF
DISEASE 3RD EDITION

17. Which of the following processes associated with cellular injury is most likely to
be reversible?
A) Cell damage resulting from accumulation of fat in the cytoplasm
B) Cellular changes as a result of ionizing radiation
C) Cell damage from accumulation of free radicals
D) Apoptosis
18. The extrinsic pathway of apoptosis can be initiated by: A)
damage to cellular DNA.

B) decreased ATP levels.


C) activation of the p53 protein.
D) activation of death receptors on the cell surface.
19. A patient with severe peripheral vascular disease has developed signs of dry
gangrene on the great toe of one foot. Which of the following pathophysiologic
processes most likely contributed to this diagnosis?
A) Inappropriate activation of apoptosis
B) Bacterial invasion
C) Impaired arterial blood supply
D) Metaplastic cellular changes
20. Which of the following facts underlies the concept of replicative senescence?
A) Genes controlling longevity are present or absent in varying quantities
among different individuals.
B) Telomeres become progressively shorter in successive generations of a
cell.
C) The damaging influence of free radicals increases exponentially in later
generations of a cell.
D) Aging produces mutations in DNA and deficits in DNA repair.

Answer Key

1. A
APPLIED PATHOPHYSIOLOGY A CONCEPTUAL APPROACH TO THE MECHANISMS OF
DISEASE 3RD EDITION

2. C
3. B
4. C
5. A
6. B
7. C
8. B
9. B
10. A
11. C
12. A
13. D
14. C
15. B
16. B
17. A
18. D
19. C
20. B

Chapter 3 Inflammation and Tissue Repair

1. The characteristic, localized cardinal signs of acute inflammation include:


A) fever.
B) fatigue.
C) redness.
D) granuloma.
2. The vascular, hemodynamic stage of acute inflammation is initiated by
momentary vasoconstriction followed by vasodilation that causes localized:
APPLIED PATHOPHYSIOLOGY A CONCEPTUAL APPROACH TO THE MECHANISMS OF
DISEASE 3RD EDITION

A) bleeding.
B) congestion.
C) pale skin.
D) coolness.
3. The cellular stage of acute inflammation is marked by the movement of
leukocytes into the area. Which of the following cells arrives early in great
numbers?
A) Basophils
B) Lymphocytes
C) Neutrophils
D) Platelets
4. The phagocytosis process involves three distinct steps. What is the initial step in
the process?
A) Engulfment
B) Intracellular killing
C) Antigen margination
D) Recognition and adherence
5. Which of the following mediators of inflammation causes increased capillary
permeability and pain?
A) Serotonin
B) Histamine
C) Bradykinin
D) Nitric oxide
6. Inflammatory exudates are a combination of several types. Which of the
following exudates is composed of enmeshed necrotic cells?
A) Serous
B) Fibrinous
C) Suppurative
D) Membranous
APPLIED PATHOPHYSIOLOGY A CONCEPTUAL APPROACH TO THE MECHANISMS OF
DISEASE 3RD EDITION

7. The acute-phase systemic response usually begins within hours of the onset of
inflammation and includes: A) fever and lethargy.

B) decreased C-reactive protein.


C) positive nitrogen balance.
D) low erythrocyte sedimentation rate.
8. In contrast to acute inflammation, chronic inflammation is characterized by
which of the following phenomena?
A) Profuse fibrinous exudation
B) A shift to the left of granulocytes
C) Metabolic and respiratory alkalosis
D) Lymphocytosis and activated macrophages
9. Exogenous pyrogens (interleukin-1) and the presence of bacteria in the blood
lead to the release of endogenous pyrogens that: A) stabilize thermal control in
the brain.

B) produce leukocytosis and anorexia.


C) block viral replication in cells.
D) inhibit prostaglandin release.
10. An older adult patient has just sheared the skin on her elbow while attempted to
boost herself up in bed, an event that has precipitated acute inflammation in the
region surrounding the wound. Which of the following events will occur during
the vascular stage of the patients inflammation?
A) Outpouring of exudate into interstitial spaces
B) Chemotaxis
C) Accumulation of leukocytes along the epithelium
D) Phagocytosis of cellular debris
11. Which of the following individuals most likely has the highest risk of
experiencing chronic inflammation?
A) A patient who has recently been diagnosed with type 2 diabetes
B) A patient who is a carrier of an antibiotic-resistant organism
APPLIED PATHOPHYSIOLOGY A CONCEPTUAL APPROACH TO THE MECHANISMS OF
DISEASE 3RD EDITION

C) A patient who is taking oral antibiotics for an upper respiratory infection


D) A patient who is morbidly obese and who has a sedentary lifestyle
12. Which of the following core body temperatures is within normal range?
A) 35.9C (96.6F)
B) 38.0C (100.4F)
C) 35.5C (95.9F)
D) 37.3C (99.1F)
13. A postsurgical patient who is recovering in the postanesthetic recovery unit states
that she is freezing cold. Which of the following measures is likely to be
initiated in the patients hypothalamus in an effort to reduce heat loss?
A) Opening of arteriovenous (AV) shunts
B) Reduced exhalation of warmed air
C) Contraction of pilomotor muscles
D) Decreased urine production
14. An elderly patient is dressed only in a hospital gown and complains of a draft in
her room. Consequently, she has requested a warm blanket while she sits in her
wheelchair. Which of the following mechanisms of heat loss is most likely the
primary cause of her request?
A) Evaporation and conduction
B) Radiation and convection
C) Conduction and convection
D) Convection and evaporation
15. Which of the following pathophysiologic processes are capable of inducing the
production of pyrogens? Select all that apply.
A) Acute inflammation
B) Obesity
C) Myocardial infarction
D) Malignancy
E) Renal failure
APPLIED PATHOPHYSIOLOGY A CONCEPTUAL APPROACH TO THE MECHANISMS OF
DISEASE 3RD EDITION

16. Which of the following patients is most likely to be susceptible to developing a


neurogenic fever?
A) A patient who has stage II Alzheimer disease
B) A patient who has sustained a head injury in a bicycle crash
C) A patient who has become delirious after the administration of a
benzodiazepine
D) A patient who has begun taking a selective serotonin-reuptake inhibitor
(SSRI) for the treatment of depression
17. Patients are commonly administered antipyretics when their oral temperature
exceeds 37.5C (99.5F). Which of the following statements related to the
rationale for this action is most accurate?
A) Temperatures in excess of 37.5C (99.5F) can result in seizure activity.
B) Lower temperatures inhibit the protein synthesis of bacteria.
C) There is little empirical evidence for this treatment modality.
D) Most common antipyretics have been shown to have little effect on core
temperature.
18. A patient has sought care because of recent malaise and high fever. Upon
assessment, the patient states that his current fever began two days earlier,
although he states that for the last 2 weeks he is in a cycle of high fever for a
couple of days followed by a day or two of normal temperature. Which of the
following fever patterns is this patient experiencing?
A) Recurrent fever
B) Remittent fever
C) Sustained fever
D) Intermittent fever
19. A febrile, 3-week-old infant has been brought to the emergency department by his
parents and is currently undergoing a diagnostic workup to determine the cause
of his fever. Which of the following statements best conveys the rationale for
this careful examination?
A) The immature hypothalamus is unable to perform normal thermoregulation.
B) Infants are susceptible to serious infections because of their decreased
immune function.
APPLIED PATHOPHYSIOLOGY A CONCEPTUAL APPROACH TO THE MECHANISMS OF
DISEASE 3RD EDITION

C) Commonly used antipyretics often have no effect on the core temperature


of infants.
D) Fever in neonates is often evidence of a congenital disorder rather than an
infection.
20. An 84-year-old patients blood cultures have come back positive, despite the fact
that his oral temperature has remained within normal range. Which of the
following phenomena underlies the alterations in fever response that occur in
the elderly?
A) Disturbance in the functioning of the thermoregulatory center
B) Increased heat loss by evaporation
C) The presence of comorbidities that are associated with lowered core
temperature
D) Persistent closure of arteriovenous shunts

Answer Key

1. C
2. B
3. C
4. D
5. C
6. D
7. A
8. D
9. B
10. A
11. D
12. D
13. C
14. B
APPLIED PATHOPHYSIOLOGY A CONCEPTUAL APPROACH TO THE MECHANISMS OF
DISEASE 3RD EDITION

15. A, C, D
16. B
17. C
18. D
19. B
20. A

Chapter 4 Altered Immunity

1. The mediators involved in type I hypersensitivity allergic responses are released


from:
A) mast cells.
B) plasma cells.
C) monocytes.
D) arachidonic acid.
2. A genetically determined hypersensitivity to common environmental allergens
causes reactions, such as: A) atopic; urticaria.

B) autoimmune; diarrhea.
C) IgM-mediated; infections.
D) delayed; poison ivy rash:
3. Mismatched blood transfusion reaction with hemolysis of blood cells is an
example of type II, mediated hypersensitivity reaction.
A) T-cell
B) antibody
C) leukotriene
D) complement
4. Type III hypersensitivity immune responses can be harmful when immune
complex deposits in tissue activate that can directly damage area
tissues.
A) inflammation
APPLIED PATHOPHYSIOLOGY A CONCEPTUAL APPROACH TO THE MECHANISMS OF
DISEASE 3RD EDITION

B) autoantibodies
C) cytotoxic cells
D) immunoglobulins
5. The mechanism by which humans recognize self-cells from non-self
(antigens)cells is .
A) autoimmunity
B) self-tolerance
C) non-self anergy
D) immunocompatibility
6. Organ rejection is a complication of organ transplantation caused by recipient
immune cells:
A) destroying the host T cells.
B) attack on the donor cells.
C) combining with grafts HLA.
D) being recognized as foreign.
7. The leading cause of death for people with HIV is opportunistic .
A) leukemia
B) tuberculosis
C) pneumonia
D) toxoplasmosis
8. Wasting syndrome, an AIDS-defining illness, is characterized by involuntary
weight loss of at least 10% of baseline body weight in the presence of:
A) diarrhea.
B) hypermetabolism.
C) weakness and fever.
D) glucose intolerance.
9. The window period of HIV infection refers to the period of time between
infection and:
APPLIED PATHOPHYSIOLOGY A CONCEPTUAL APPROACH TO THE MECHANISMS OF
DISEASE 3RD EDITION

A) transmission.
B) seroconversion.
C) initial symptoms.
D) antibody screening.
10. HIV-positive persons that display manifestations of laboratory category 3 or
clinical category C are considered to have: A) zero viral load.

B) seroconversion.
C) complete remission.
D) AIDS-defining illnesses.
11. Contact with poison ivy has resulted in intense pruritus, erythema, and weeping
on a patients forearm. Which of the following processes resulted in the patients
signs and symptoms?
A) IgE-mediated mast cell degranulation
B) Formation of antigen-antibody complexes
C) Cytokine release by sensitized T cells
D) Formation of antibodies against cell surface antigens
12. A patient with a long history of hay fever has recently begun a series of
immunotherapy (allergy shots). How will this treatment potentially achieve a
therapeutic effect?
A) By blocking cytokine release from sensitized mast cells
B) By preventing mast cells from becoming sensitized
C) By causing T cells to be sequestered in the thymus for longer periods
D) By stimulating production of IgG to combine with antigens
13. A patient with a diagnosis of cirrhosis has experienced an acute rejection of a
donor liver. Which of the following cells is central to the rejection of the patients
transplanted organ?
A) Natural killer cells
B) Mast cells
C) T cells
APPLIED PATHOPHYSIOLOGY A CONCEPTUAL APPROACH TO THE MECHANISMS OF
DISEASE 3RD EDITION

D) Neutrophils
14. A patient with a diagnosis of aplastic anemia has undergone allogenic bone
marrow transplantation. Which of the following signs and symptoms would
most clearly suggest the existence of graft-versus-host disease (GVHD)?
A) Shortness of breath, audible crackles, and decreasing PaO2
B) Presence of a pruritic rash that has begun to slough off
C) Development of metabolic acidosis
D) Diaphoresis, fever, and anxiety
15. A patient has developed pericarditis after developing acute glomerulonephritis, a
development that may be attributable to the presence of similar epitopes on
group A, b-hemolytic streptococci and the antigens in the patients heart tissue.
Which of the following has most likely accounted for this patients autoimmune
response?
A) Breakdown of T-cell anergy
B) Release of sequestered antigens
C) Superantigens
D) Molecular mimicry
16. A 70-year-old female patient has had her mobility and independence significantly
reduced by rheumatoid arthritis. Which of the following processes likely
contributed to the development of her health problem?
A) Delayed-type hypersensitivity (DTH) reaction
B) Proliferation of cytotoxic T cells
C) Failure of normal self-tolerance
D) Deletion of autoreactive B cells
17. Which of the following would constitute a normal assessment finding in a
neonate?
A) Minimal or absent levels of IgA and IgM
B) Absence of plasma cells in the lymph nodes and spleen
C) Undetectable levels of all immunoglobulins
D) Absence of mature B cells with normal T-cell levels and function
APPLIED PATHOPHYSIOLOGY A CONCEPTUAL APPROACH TO THE MECHANISMS OF
DISEASE 3RD EDITION

18. A patient was diagnosed as HIV positive several years ago. Which of the
following blood tests is most clinically useful for determining the stage and
severity of her disease?
A) Plasma levels
B) CD4+ cell counts
C) Viral load
D) White blood cell count with differential
19. A patient has been admitted to the hospital for the treatment of HIV infection,
which has recently progressed to overt AIDS. Which of the following nursing
actions should the nurse prioritize when providing care for this patient?
A) Frequent neurologic vital signs and thorough skin care
B) Hemodynamic monitoring and physical therapy
C) Careful monitoring of fluid balance and neurologic status
D) Astute infection control and respiratory assessments
20. Shortly after being diagnosed with HIV, a patient has begun highly active
antiretroviral therapy (HAART). What is the primary goal of the patients drug
regimen?
APPLIED PATHOPHYSIOLOGY A CONCEPTUAL APPROACH TO THE MECHANISMS OF
DISEASE 3RD EDITION
A)
B)
C)
D)
To limit the latent period of HIV
To slow the progression of the disease
To minimize opportunities for transmission To
prevent seroconversion

Answer Key

1. A
2. A
3. B
4. A
5. B
6. B
7. B
8. A
9. B
10. D
11. C
12. D
13. C
14. B
15. D
16. C
17. A
18. B
APPLIED PATHOPHYSIOLOGY A CONCEPTUAL APPROACH TO THE MECHANISMS OF
DISEASE 3RD EDITION

19. D
20. B

Chapter 5 Infection

1. Although growth rate is variable among types of bacteria, the growth of bacteria
is dependent on:
A) biofilm communication.
B) availability of nutrients.
C) an intact protein
capsid.

D) individual cell motility.


2. Treponema pallidum, the cause of syphilis, is a spirochete bacterium that is
spread from human to human by: A) tick or lice vector bites.

B) direct physical contact.


C) exposure to infected urine.
D) inhaling airborne particles.
3. Chlamydiaceae, a rather common sexually transmitted infectious organism, has
characteristics of both viruses and bacteria. The infectious form of this
organisms life cycle is until it enters the host cell.
A) an elementary body
B) adhered to cholesterol
C) propelled by filaments
D) encapsulated hyphae
4. Because dermatophytes are capable of growing , the infection is
mainly found on cutaneous surfaces of the body.
A) a powdery colony
B) in moist skin folds
C) on cooler tissue
APPLIED PATHOPHYSIOLOGY A CONCEPTUAL APPROACH TO THE MECHANISMS OF
DISEASE 3RD EDITION
A)
B)
C)
D)
D) branching filaments
5. Although both eukaryotes and prokaryotes are capable of causing infectious
diseases in humans, eukaryotes are unique because they have a distinct:
organized nucleus. circular plasmid DNA. cytoplasmic membrane.
variation of shape and size.
6. Whatever the mechanism of entry, the human-to-human transmission of
infectious agents is directly related to the:
A) source of contact.
B) site of infection.
C) number of pathogens absorbed.
D) virulence factors.
7. The course of any infectious disease progresses through several distinct stages
after the pathogen enters the host. Although the duration may vary, the
hallmark of the prodromal stage is:
A) tissue inflammation and damage.
B) initial appearance of symptoms.
C) progressive pathogen elimination.
D) containment of infectious pathogens.
8. Although bacterial toxins vary in their activity and effects on host cells, a small
amount of gram-negative bacteria endotoxin: A) is released during cell growth.

B) inactivates key cellular functions.


C) uses protein to activate enzymes.
D) in the cell wall activates inflammation.
9. Serology testing includes the measurement of which of the following?
A) Antibody titers
B) Culture growth
APPLIED PATHOPHYSIOLOGY A CONCEPTUAL APPROACH TO THE MECHANISMS OF
DISEASE 3RD EDITION

C) Direct antigens
D) DNA sequencing
10. Prions cause transmissible neurodegenerative diseases and are characterized by:
A) a lack of reproductive capacity.

B) hypermetabolism.
C) enzyme production.
D) chronic inflammation.
11. Which of the following individuals is experiencing a health problem that is the
result of a parasite?
A) A college student who contracted Chlamydia trachomatis during an
unprotected sexual encounter
B) A man who acquired malaria while on a tropical vacation
C) A hospital patient who has developed postoperative pneumonia
D) A woman who developed hepatitis A from eating at an unhygienic
restaurant
12. Which of the following traits is characteristic of saprophytes? A)
They derive energy from decaying organic matter.

B) They are beneficial components of human microflora.


C) They have RNA or DNA, but never both.
D) They are capable of spore production.
13. A hospital patient was swabbed on admission for antibiotic-resistant organisms
and has just been informed that methicillin-resistant Staphylococcus aureus
(MRSA) is present in his groin. The patient has a normal core temperature and
white blood cell count. This patient is experiencing which of the following?
A) Infection
B) Proliferation
C) Colonization
D) Inflammation
APPLIED PATHOPHYSIOLOGY A CONCEPTUAL APPROACH TO THE MECHANISMS OF
DISEASE 3RD EDITION
A)
B)
C)
D)
14. A 33-year-old patient who is a long-term intravenous user of heroin has been
recently diagnosed with hepatitis C. Which of the following portals of entry
most likely led to the patients infection?
Direct contact
Vertical transmission
Ingestion
Penetration
15. A 9-month-old infant has been diagnosed with botulism after he was fed honey.
The childs mother was prompted to seek care because of this childs sudden
onset of neuromuscular deficits, which were later attributed to the release of
substances by Clostridium botulinum bacteria. Which virulence factor
contributed to this childs illness?
A) Endotoxins
B) Adhesion factors
C) Exotoxins
D) Evasive factors
16. A patient with a long-standing diagnosis of Crohn disease has developed a
perianal abscess. Which of the following treatments will this patient most likely
require?
A) Antiviral therapy
B) Antibiotic therapy
C) Surgical draining
D) Pressure dressing
17. A patients primary care provider has ordered direct antigen detection in the care
of a patient with a serious symptomatology of unknown origin. Which of the
following processes will be conducted?
A) Detecting DNA sequences that are unique to the suspected pathogen
B) Growth of biofilms on various media in the laboratory setting
APPLIED PATHOPHYSIOLOGY A CONCEPTUAL APPROACH TO THE MECHANISMS OF
DISEASE 3RD EDITION

C) Quantification of IgG and IgM antibodies in the patients blood


D) Introduction of monoclonal antibodies to a blood sample from the patient
18. A patient has begun taking acyclovir, an antiviral medication, to control herpes
simplex outbreaks. What is this drugs mechanism of action?
A) Inhibition of viral adhesion to cells
B) Elimination of exotoxin production
C) Antagonism of somatic cell binding sites
D) Interference with viral replication processes
19. International travel has contributed to increased prevalence and incidence of
nonindigenous diseases by increasing which of the following?
A) Portals of entry
B) Sources of infection
C) Virulence
D) Disease course
20. A public health nurse should recognize that sexually transmitted infections
(STIs) are typically spread by which of the following mechanisms?
A) Penetration
B) Vertical transmission
C) Direct contact
D) Ingestion

Answer Key

1. B
2. B
3. A
4. C
5. A
6. C
APPLIED PATHOPHYSIOLOGY A CONCEPTUAL APPROACH TO THE MECHANISMS OF
DISEASE 3RD EDITION
A)
B)
C)
D)
7. B
8. D
9. A
10. A
APPLIED PATHOPHYSIOLOGY A CONCEPTUAL APPROACH TO THE MECHANISMS OF
DISEASE 3RD EDITION

11. B
12. A
13. C
14. D
15. C
16. C
17. D
18. D
19. B
20. C

Chapter 6 Genetic and Developmental Disorders

1. Genetic disorders that involve a single gene trait are characterized by: A)
multifactorial gene mutations.

B) chromosome rearrangements.
C) Mendelian patterns of transmission.
D) abnormal numbers of chromosomes.
2. In addition to having a 50% chance of inheriting an autosomal dominant
disorder from an affected parent, such a disorder is characterized by: A)
aneuploidy of genes in all cells.

B) deficiencies in enzyme synthesis.


C) affected X transmission to daughters.
D) varied gene penetration and expression.
3. Autosomal recessive disorders are characterized by: A)
age of onset later in life.

B) abnormal protein structure.


C) inborn errors of metabolism.
APPLIED PATHOPHYSIOLOGY A CONCEPTUAL APPROACH TO THE MECHANISMS OF
DISEASE 3RD EDITION

D) one in two risk of a carrier child.


4. When a male child inherits an X-linked disorder from his heterozygous carrier
mother,
A) his sons will be carriers.
B) his father has the disorder.
C) some of his sisters will be carriers.
D) his daughters will have the disorder.
5. Multifactorial inheritance disorders, such as cleft palate, are often caused
by during fetal development.
A) multiple gene mutations
B) dominant gene expression
C) X-linked crossover problem
D) polyploidy of chromosomes
6. The newborn has the distinctive physical features of trisomy 21, Down
syndrome, which includes:
A) upward slanting of eyes.
B) large, protruding ears.
C) thin lips and small tongue.
D) long fingers with extra creases.
7. Aneuploidy of the X chromosome can result in a monosomy or polysomy
disorder. The manifestations of monosomy X, Turner syndrome, differ from
polysomy X disorders in numerous ways that include: A) short-stature female
individual..
B) mental retardation.
C) enlarged breasts.
D) early onset puberty.
8. A teratogenic environmental agent can cause birth defects when: A)
inherited as a recessive trait.
B) intense exposure occurs at birth.
APPLIED PATHOPHYSIOLOGY A CONCEPTUAL APPROACH TO THE MECHANISMS OF
DISEASE 3RD EDITION

C) disjunction occurs during meiosis.


D) retained during early pregnancy.
9. Fetal alcohol syndrome (FAS) is unlike other teratogens in that the harmful
effects on the fetus:
A) directly result in liver damage.
B) extend throughout the pregnancy.
C) is most noticeable in adulthood.
D) cause death in early childhood.
10. Prenatal diagnosis methods include the use of ultrasonography for identifying
abnormalities.
A) cytogenic
B) skeletal
C) chromosomal
D) a-fetoprotein
11. A woman who is a carrier for which of the following diseases possesses the
greatest likelihood of passing the disease to her future children when
heterozygous pairing exists?
A) Phenylketonuria (PKU)
B) Tay-Sachs disease
C) Neurofibromatosis
D) Cystic fibrosis
12. Which of the following statements is true of autosomal recessive disorders? A)
Onset is typically late in childhood or early in adulthood.

B) Symptomatology is less uniform than with autosomal dominant disorders.


C) Mitochondrial DNA is normally the site of genetic alteration.
D) Effects are typically the result of alterations in enzyme function.
13. The parents of a newborn infant are relieved that their baby was born healthy,
with the exception of a cleft lip that will be surgically corrected in 10 or 12
APPLIED PATHOPHYSIOLOGY A CONCEPTUAL APPROACH TO THE MECHANISMS OF
DISEASE 3RD EDITION

weeks. Which of the nurses following statements to the parents best conveys
the probable cause of the infants cleft lip?
A) Though you are both healthy, you likely both carry the gene for a cleft lip.
B) Provided one of you had the gene for a cleft lip, your baby likely faced a
50/50 chance of having one.
C) Your childs cleft lip likely results from the interplay between environment
and genes.
D) A cleft lip can sometimes result from taking prescription drugs, even when
theyre taken as ordered.
14. Which of the following practitioners is most likely to be of assistance in the
early care of an infant with a cleft lip?
A) Lactation consultant
B) Respiratory therapist
C) Occupational therapist
D) Social worker
15. A 41-year-old woman has made the recent decision to start a family, and is eager
to undergo testing to mitigate the possibility of having a child with Down
syndrome. Which of the following tests is most likely to provide the data the
woman seeks?
A) Genetic testing of the woman
B) Genetic testing of the woman and the father
C) Prenatal blood tests
D) Ultrasonography
16. Genetic testing has revealed that a male infant has been born with an extra X
chromosome. What are the most likely implications of this finding?
A) The child is unlikely to survive infancy
B) The child is likely to have no manifestations of this chromosomal
abnormality
C) The child will have significant neurological and cognitive defects
D) The child will be unable to reproduce
APPLIED PATHOPHYSIOLOGY A CONCEPTUAL APPROACH TO THE MECHANISMS OF
DISEASE 3RD EDITION

17. Which of the following variables determine the extent of teratogenic drug
effects? Select all that apply.
A) Maternal health history
B) Molecular weight of the drug
C) Stage of pregnancy when the drug was taken
D) Duration of drug exposure
E) Fetal blood type
18. A woman who has just learned that she is pregnant for the first time has sought
advice from her healthcare provider about the safe use of alcohol during
pregnancy. What advice should the clinician provide to the woman?
A) Its likely best to eliminate alcohol from your diet while youre pregnant.
B) Moderation in alcohol use is critical while you are pregnant.
C) You should limit yourself to a maximum of one drink daily while youre
pregnant.
D) You should drink no alcohol until you are in your second trimester.
19. Which of the following health problems may be identified by a TORCH
screening test?
A) Rubella and herpes
B) Tenovaginitis and human papillomavirus
C) Rhinovirus and Ormond disease
D) Chlamydia and rickets
20. Ultrasonography is most likely to detect which of the following fetal
abnormalities?
A) Neural tube defects
B) Skeletal abnormalities
C) Chromosomal defects
D) Single-gene disorders
Answer Key

1. C
APPLIED PATHOPHYSIOLOGY A CONCEPTUAL APPROACH TO THE MECHANISMS OF
DISEASE 3RD EDITION

2. D
3. C
4. C
5. A
6. A
7. A
8. D
9. B
10. B
11. C
12. D
13. C
14. A
15. C
16. B
17. B, C, D
18. A
19. A
20. B

Chapter 7 Altered Cellular Proliferation and Differentiation

1. Epithelialization, the first component of the proliferative phase of wound


healing, is delayed in open wounds until after has formed.
A) granulation tissue
B) fibrinous meshwork
C) capillary circulation
D) collagenous layers
APPLIED PATHOPHYSIOLOGY A CONCEPTUAL APPROACH TO THE MECHANISMS OF
DISEASE 3RD EDITION

2. A mutation has occurred during mitosis of an individuals bone marrow cell. This
event may be the result of the failure of which of the following?
A) Progenitor cells
B) Fibroblasts
C) Stem cells
D) Cyclins
3. A patient has experienced a myocardial infarction with accompanying necrosis
of cardiac muscle, a permanent tissue. What are the ramifications of the fact
that cardiac muscle is a permanent tissue?
A) The cardiac muscle cells will remain perpetually in the G1 stage of mitosis.
B) Regeneration of the patients cardiac muscle will be exceptionally slow.
C) The necrotic cells will be replaced with muscle cells that have limited
metabolism.
D) The cells will not proliferate and will be replaced with scar tissue.
4. A couple have chosen to pay for the harvesting and storage of umbilical cord
blood after the delivery of their child to secure a future source of embryonic
stem cells. What is the most likely rationale for the couples decision?
A) The stem cells may be able to produce a wide range of body cells.
B) The embryonic stem cells allow stable and permanent tissues to enter
mitosis.
C) The stem cells can change the proliferative capacity of other cells.
D) The embryonic stem cells remove cyclin-dependent kinase inhibitors from
the body.
5. The basement membrane surrounding a patients foot wound remains intact, a
fact that bodes well for the wound-healing process. Which of the following
components constitute this form of the extracellular matrix? Select all that
apply.
A) Prostaglandins
B) Fibrous structural proteins
C) Lymphocytes
D) Water-hydrated gels
APPLIED PATHOPHYSIOLOGY A CONCEPTUAL APPROACH TO THE MECHANISMS OF
DISEASE 3RD EDITION

E) Glycoproteins
6. A nursing student is cleaning and changing the dressing on a patients sacral
ulcer. The student has vigorously cleansed the wound bed to remove all traces
of the beefy, red tissue that existed in the wound bed. The student has most
likely removed:
A) Necrotic tissue
B) Granulation tissue
C) Stem cells
D) The extracellular matrix
7. A 12-year-old boys severe wound that he received from a dog bite has begun to
heal and currently shows no signs of infection. Which of the following
processes occurred first during this process of repair by connective tissue
deposition?
A) Reorganization of fibrous tissue
B) Angiogenesis
C) Emigration of fibroblasts to the wound site
D) Deposition of the extracellular matrix
8. Which of the following wounds is most likely to heal by secondary intention?
A) A finger laceration that a cook received while cutting up onions
B) A boys road rash that he got by falling off his bicycle
C) A needlestick injury that a nurse received when injecting a patients
medication
D) The incision from a teenagers open appendectomy
9. A patient underwent an open cholecystectomy 4 days ago and her incision is
now in the proliferative phase of healing. What is the dominant cellular process
that characterizes this phase of the patients healing?
A) Hemostasis and vasoconstriction
B) Keloid formation
C) Collagen secretion by fibroblasts
D) Phagocytosis by neutrophils
APPLIED PATHOPHYSIOLOGY A CONCEPTUAL APPROACH TO THE MECHANISMS OF
DISEASE 3RD EDITION

10. Which of the following surgical patients is most likely to experience enhanced
wound healing as a result of his or her diet?
A) A patient who eats a high-calorie diet and large amounts of red meat
B) A patient who is a vegetarian and who eats organic foods whenever
possible
C) A patient who practices carefully calorie control and who avoids animal
fats
D) A patient who is receiving total parenteral nutrition due to recurrent nausea
11. Which of the following patients is most likely to experience impaired wound
healing?
A) A patient with a diagnosis of type 1 diabetes and a history of poor blood
sugar control
B) A child whose severe cleft lip and palate have required a series of
surgeries over several months
C) A patient who takes nebulized bronchodilators several times daily to treat
chronic obstructive pulmonary disease
D) A patient with persistent hypertension who takes a b-adrenergic blocker
and a potassium-wasting diuretic daily

Answer Key

1. A
2. D
3. D
4. A
5. B, D, E
6. B
7. B
8. B
9. C
10. A
APPLIED PATHOPHYSIOLOGY A CONCEPTUAL APPROACH TO THE MECHANISMS OF
DISEASE 3RD EDITION

11. A

Chapter 8 Altered Fluid, Electrolyte, and Acid-Base Balance

1. An injured patient develops interstitial edema as a result of decreased:


A) vascular volume.
B) hydrostatic pressure.
C) capillary permeability.
D) colloidal osmotic pressure.
2. The most reliable method for measuring body water or fluid volume increase is
by assessing:
A) tissue turgor.
B) intake and output.
C) body weight change.
D) serum sodium levels.
3. The syndrome of inappropriate ADH is characterized by: A)
increased osmolality.

B) excessive water thirst.


C) copious dilute urination.
D) dilutional hyponatremia.
4. In isotonic fluid volume deficit, changes in total body water are accompanied
by:
A) intravascular hypotonicity.
B) increased intravascular water.
C) increases in intracellular sodium.
D) proportionate losses of sodium.
5. Hyponatremia can be caused by and manifested by .
A) hypovolemia; dehydration
B) third spacing; hypertonicity
APPLIED PATHOPHYSIOLOGY A CONCEPTUAL APPROACH TO THE MECHANISMS OF
DISEASE 3RD EDITION

C) water retention; hypotonicity


D) aldosterone excess; low ADH
6. One of the major causes of hyperkalemia is , which alters
potassium elimination.
A) renal dysfunction
B) aldosterone excess
C) metabolic alkalosis
D) plasma albumin deficit
7. Hypoparathyroidism causes hypocalcemia by: A)
increasing serum magnesium.

B) increasing phosphate excretion.


C) blocking bone release of calcium.
D) blocking action of intestinal vitamin D.
8. Magnesium is important for the overall function of the body because of its direct
role in:
A) cell membrane permeability.
B) somatic cell growth control.
C) sodium and tonicity regulation.
D) DNA replication and transcription.
9. A patient has acidosis that is suspected to be respiratory in etiology. Which of
the following is the major cause of acute primary respiratory acidosis?
A) Decreased CO2 retention
B) Increased metabolic acids
C) Renal bicarbonate retention
D) Impaired alveolar ventilation
10. As other mechanisms prepare to respond to a pH imbalance, immediate
buffering is a result of increased: A) intracellular albumin.
B) hydrogen/potassium binding.
APPLIED PATHOPHYSIOLOGY A CONCEPTUAL APPROACH TO THE MECHANISMS OF
DISEASE 3RD EDITION

C) sodium/phosphate anion absorption.


D) bicarbonate/carbonic acid regulation.
11. A patient with a diagnosis of liver cirrhosis secondary to alcohol use has a
distended abdomen as a result of fluid accumulation in his peritoneal cavity
(ascites). Which of the following pathophysiologic processes contributes to this
third spacing?
A) Abnormal increase in transcellular fluid volume
B) Increased capillary colloidal osmotic pressure
C) Polydipsia
D) Impaired hormonal control of fluid volume
12. A patient has been receiving intravenous normal saline at a rate of 125 mL per
hour since her surgery 2 days earlier. As a result of her consequent increase in
vascular volume, she has become edematous. Which of the following
phenomena accounts for this patients edema?
A) Obstruction of lymph flow
B) Increased capillary permeability
C) Decreased capillary colloidal osmotic pressure
D) Increased capillary filtration pressure
13. A patient with a diagnosis of schizophrenia has been admitted to the emergency
department after ingesting more than 2 gallons of water. Which of the following
pathophysiologic processes may result from the sudden water gain?
A) Hypernatremia
B) Water movement from the extracellular to intracellular compartment
C) Syndrome of inappropriate secretion of ADH (SIADH)
D) Isotonic fluid excess in the extracellular fluid compartment
14. Which of the following patients would likely be at highest risk of developing
hyperkalemia?
A) A patient who has been admitted for the treatment of acute renal failure
following a drug overdose
B) A patient who has experienced an ischemic stroke with multiple sensory
and motor losses
APPLIED PATHOPHYSIOLOGY A CONCEPTUAL APPROACH TO THE MECHANISMS OF
DISEASE 3RD EDITION

C) An elderly patient who is experiencing vomiting and diarrhea as a result of


influenza
D) A patient whose thyroidectomy resulted in the loss of his parathyroid
gland
15. A female patient with a history of chronic renal failure has developed
hypocalcemia. Which of the following assessment findings would provide
potential confirmation of this diagnosis?
A) The patient experiences shortness of breath on exertion with decreased
oxygen saturation levels.
B) The patient is difficult to rouse and is disoriented to time and place.
C) The patients heart rate is 120 beats per minute and she is diaphoretic
(sweaty).
D) The patient has muscle spasms and complains of numbness around her
mouth.
16. Which of the following assessments should be prioritized in the care of a patient
who is being treated for hypokalemia?
A) Detailed fluid balance monitoring
B) Arterial blood gases
C) Cardiac monitoring
D) Monitoring of hemoglobin levels and oxygen saturation
17. Magnesium is an important component of which of the following processes that
are integral to the maintenance of homeostasis? Select all that apply.
A) Intracellular and extracellular buffering
B) Cellular energy metabolism
C) Function of the sodium-potassium pump
D) Nerve conduction
E) Cell membrane function
18. A 77-year-old woman has been brought to the emergency department by her
daughter because of a sudden and unprecedented onset of confusion. The
patient admits to ingesting large amounts of baking soda since the morning in
APPLIED PATHOPHYSIOLOGY A CONCEPTUAL APPROACH TO THE MECHANISMS OF
DISEASE 3RD EDITION

an effort to treat indigestion. How will the womans body attempt to resolve this
disruption in acid-base balance?
A) Hyperventilation
B) Increasing renal H+ excretion
C) Increased renal HCO3 reabsorption
D) Hypoventilation
19. Arterial blood gases of a patient with a diagnosis of acute renal failure reveal a
pH of 7.25 (low), HCO3-of 21 mEq/L (low), decreased PCO2 accompanied by a
respiratory rate of 32 (high). What disorder of acid-base balance is the patient
most likely experiencing?
A) Metabolic acidosis
B) Metabolic alkalosis
C) Respiratory acidosis
D) Respiratory alkalosis
20. A nurse who is providing care for a patient with a diagnosis of diabetes insipidus
should prioritize the close monitoring of serum levels of which of the following
electrolytes?
A) Potassium
B) Sodium
C) Magnesium
D) Calcium

Answer Key

1. D
2. C
3. D
4. D
5. C
6. A
APPLIED PATHOPHYSIOLOGY A CONCEPTUAL APPROACH TO THE MECHANISMS OF
DISEASE 3RD EDITION

7. C
8. D
9. D
10. D
11. A
12. D
13. B
14. A
15. D
16. C
17. B, C, D, E
18. D
19. A
20. B

Chapter 9 Altered Neuronal Transmission

1. The somatic nervous system provides sensory and motor innervation for: A)
peripheral nerves.

B) abdominal viscera.
C) secretory glands.
D) smooth muscle.
2. The proteins and other materials used by the axon are synthesized and
then flow down the axon through its cytoplasm.
A) in the cell body
B) by Nissl bodies
C) through dendrites
D) across synapses
APPLIED PATHOPHYSIOLOGY A CONCEPTUAL APPROACH TO THE MECHANISMS OF
DISEASE 3RD EDITION

3. Supporting cells of the nervous system, such as Schwann cells, satellite cells,
and types of glial cells, function to provide neurons with: A) local protection.

B) control functions.
C) membrane permeability.
D) integrative metabolism.
4. Neurons are characterized by the ability to communicate with other neurons and
body cells through:
A) astrocytes.
B) axon hillocks.
C) nodes of Ranvier.
D) action potentials.
5. Chemical synapses rely on in order to provide communication between
neurons.
A) diffusion
B) gap junctions
C) satellite cells
D) transmitter molecules
6. The blood-brain and CSF-brain barriers control the chemical environment of the
brain by allowing easy entrance to only a few chemicals that include:
A) oxygen.
B) protein.
C) glutamate.
D) potassium.
7. The perception of where a stimulus is in space and in relation to body parts is a
function of the:
A) occipital lobe.
B) parietal lobe.
C) hypothalamus.
APPLIED PATHOPHYSIOLOGY A CONCEPTUAL APPROACH TO THE MECHANISMS OF
DISEASE 3RD EDITION

D) prefrontal cortex.
8. The pia mater is a connective tissue sheath that covers the spinal cord and also
contains:
A) spinal fluid.
B) fibrocartilage.
C) blood
vessels.

D) segmental nerves.
9. Which of the following is the neurotransmitter for most postganglion
sympathetic neurons?
A) Enkephalin
B) Glutamic acid
C) Catecholamines
D) Acetylcholine
10. In contrast to the sympathetic nervous system, the functions of the
parasympathetic nervous system include:
A) sweating.
B) anabolism.
C) pupil dilation.
D) vasoconstriction.
11. Which of the following substances provides the majority of the fuel needs of the
neurologic system?
A) Glycogen
B) Glucose
C) Amino acids
D) Triglycerides
12. A 60-year-old woman has been recently diagnosed with multiple sclerosis, a
disease in which the oligodendrocytes of the patients central nervous system
APPLIED PATHOPHYSIOLOGY A CONCEPTUAL APPROACH TO THE MECHANISMS OF
DISEASE 3RD EDITION

(CNS) are progressively destroyed. Which physiologic process within the


neurologic system is most likely be affected by this disease process?
A) Oxygen metabolism
B) Neurotransmitter synthesis
C) Nerve conduction
D) Production of cerebrospinal fluid
13. A neuron has been hyperpolarized. How will this affect the excitability of the
neuron?
A) The neuron will have a membrane potential farther from the threshold.
B) The neuron will be more difficult to repolarize after firing.
C) The membrane potential of the neuron will be closer to the threshold.
D) The neurons excitability will be significantly increased.
14. A pregnant womans most recent ultrasound is suggestive of spina bifida, and her
primary care provider has subsequently order further diagnostic testing. The
pathophysiologic effects of this disease are due to: A) malformation of the
mesoderm.
B) abnormal closure of the neural tube.
C) lesions in the dorsal root ganglia.
D) hypertrophy of the primary vesicles.
15. Which of the following messages is most likely to be carried by general somatic
afferent (GSA) neurons?
A) The sensation of cold when touching ice
B) The message to move a finger and thumb
C) The message to move the larynx during speech
D) Information about the position of a joint
16. Which of the following processes is most likely to occur as a result of a spinal
reflex?
A) Peristalsis of the small and large bowel
B) Control of oculomotor function in changing light levels
APPLIED PATHOPHYSIOLOGY A CONCEPTUAL APPROACH TO THE MECHANISMS OF
DISEASE 3RD EDITION

C) Pain sensation from a potentially damaging knee movement


D) Withdrawal of a hand from a hot stove element
17. A patient has required mechanical ventilation following a traumatic head injury
sustained in a motorcycle crash, during which he sustained damage to his
respiratory center. Which of the patients brain structures has been injured?
A) Brain stem
B) Midbrain
C) Diencephalon
D) Frontal lobe
18. A patient with a diagnosis of epilepsy has required surgical removal of part of
her prefrontal cortex. Which of the following effects should her family and care
team anticipate?
A) Lapses in balance and coordination
B) Deficits in regulation of the endocrine system
C) Sensory losses
D) Changes in behavior and judgment
19. A patients primary care provider has prescribed a b-adrenergic receptor blocker.
Which of the following therapeutic effects do the patient and care provider
likely seek?
A) Reduction in heart rate and blood pressure
B) Slowing of gastrointestinal motility
C) Increase in mental acuity
D) Decreased production of gastric acid
20. Neurotrophic factors contribute to the maintenance of homeostasis in which of
the following ways?
A) By catalyzing the effects of neurotransmitters
B) By increasing the sensitivity of receptors on postsynaptic cells
C) By promoting the growth and survival of neurons
D) By selectively increasing or decreasing the release of neurotransmitters
APPLIED PATHOPHYSIOLOGY A CONCEPTUAL APPROACH TO THE MECHANISMS OF
DISEASE 3RD EDITION

Answer Key

1. A
2. A
3. A
4. D
5. D
6. A
7. B
8. C
9. C
10. B
11. B
12. C
13. A
14. B
15. A
16. D
17. A
18. D
19. A
20. C

Chapter 10 Altered Sensory Function and Pain Perception

1. The somatosensory system consists of three types of sensory neurons. The


special somatic type of afferent sensory neurons has receptors that sense:
A) muscle position.
B) visceral fullness.
APPLIED PATHOPHYSIOLOGY A CONCEPTUAL APPROACH TO THE MECHANISMS OF
DISEASE 3RD EDITION

C) temperature.
D) painful touch.
2. Proprioceptive somatosensory information from the limbs and trunk is
transmitted by the:
A) reflexive networks.
B) dorsal root ganglion neurons.
C) anterolateral pathway.
D) trigeminal sensory neurons.
3. Full localization, discrimination of intensity, and interpretation of
somatosensory stimuli requires processing by the: A) somatosensory
cortex.
B) autonomic nervous system.
C) Ruffini end-organ receptors.
D) Pacinian corpuscle receptors.
4. What pain theory proposes that pain receptors share pathways with other
sensory modalities and that different activity patterns of the same neurons can
be used to signal painful or nonpainful stimuli?
A) Pattern
B) Specificity
C) Gate control
D) Neuromatrix
5. Nociceptors are sensory receptors that are activated by:
A) cortisol.
B) noxious stimuli.
C) pressure and touch.
D) sudden movements.
6. When a person is stung on the index finger by a bee, the thalamus interprets the
pain as:
APPLIED PATHOPHYSIOLOGY A CONCEPTUAL APPROACH TO THE MECHANISMS OF
DISEASE 3RD EDITION

A) somewhere on the hand.


B) a spot on the index finger.
C) attributable to a bee stung.
D) similar to a previous bee sting.
7. Pain assessment is likely to be most challenging when providing care for which
of the following older adult patients?
A) A 90-year-old patient who takes multiple medications for cardiac and
respiratory conditions
B) A 77-year-old man who has sustained burns on the lower part of his body
C) An 82-year-old woman who has been diagnosed with diabetes and an
anxiety disorder
D) An 87-year-old man with vascular dementia and numerous other health
problems
8. In contrast to acute pain, persistent chronic pain: A)
serves as a warning system.

B) raises the pain threshold.


C) imposes physiologic stresses.
D) causes secondary reflexive spasms.
9. The sites of referred pain are determined by: A)
intensity-coding receptors.

B) location of the noxious stimuli.


C) visceral embryonic development.
D) stimulation that activates C fibers.
10. Complex regional pain syndrome is characterized by: A)
repetitious dermatome pain attacks.

B) trigeminal neuralgia with facial tics.


C) severe limb pain after amputation.
APPLIED PATHOPHYSIOLOGY A CONCEPTUAL APPROACH TO THE MECHANISMS OF
DISEASE 3RD EDITION

D) disproportionate pain with mobility.


11. An otherwise-healthy patient has been referred to a pain clinic because she
claims to experience exquisite pain from the friction of her clothes on her torso.
This patient is likely to be diagnosed with which of the following health
problems?
A) Visceral pain
B) Hypoalgesia
C) Allodynia
D) Primary hyperalgesia
12. A woman has cut her finger while dicing onions in the kitchen, causing her to
drop her knife in pain. Which of the following components of this pain signal
was transmitted by a third-order neuron?
A) The neurons between the womans finger and the womans spinal cord
B) The neurons between the thalamus and the cortex
C) The neurons between the CNS and the thalamus
D) The neurons of the efferent pathway that causes muscle contraction
13. A patient with a diagnosis of lung cancer has developed bone metastases
resulting in severe and protracted pain. Which of the following assessment
components should the nurse prioritize when assessing the patients pain?
A) The appearance of grimacing, guarding, or wincing
B) The presence of changes in vital signs that correspond to pain
C) The patients subjective report of the character and severity of pain
D) The results of a detailed neurologic assessment
14. A hospital patient has been reluctant to accept morphine sulfate despite visible
signs of pain. Upon questioning, the patient reveals that he is afraid of
becoming addicted to the drug. How can a member of the care team best
respond to the patients concern?
A) You might become addicted, but there are excellent resources available in
the hospital to deal with that development.
B) You should likely prioritize the control of your pain over any fears of
addiction that you have.
APPLIED PATHOPHYSIOLOGY A CONCEPTUAL APPROACH TO THE MECHANISMS OF
DISEASE 3RD EDITION

C) If you start needing higher doses to control your pain, then well address
those concerns.
D) Theres only a minute chance that you will become addicted to these
painkillers.
15. Which of the following patients may be experiencing the effects of neuropathic
pain?
A) A girl whose playground accident resulted in an arm fracture
B) A man with pain secondary to his poorly controlled diabetes
C) An elderly woman with a stage III pressure ulcer
D) A man whose pain is caused by gastric cancer
16. Which of the following statements is true of migraine headaches?
A) Non-pharmacologic treatments and lifestyle modifications can control
migraines in most patients.
B) Surgical treatments for migraines are indicated if pharmacologic
treatments are unsuccessful.
C) Migraines may have a hormonal etiology in some patients.
D) Opioid analgesics achieve adequate pain control in most patients.
17. Which of the following pain disorders is a manifestation of a disruption of
cranial nerve function?
A) Trigeminal neuralgia
B) Complex regional pain syndrome
C) Phantom limb pain
D) Temporomandibular joint (TMJ) syndrome
18. Which of the following characteristics differentiates a migraine with aura from a
migraine without aura?
A) Gastrointestinal involvement in the hours leading up to the headache
B) A decrease in mood and affect prior to the headache
C) Lack of response to non-pharmacologic treatments
D) Visual symptoms that precede the headache
APPLIED PATHOPHYSIOLOGY A CONCEPTUAL APPROACH TO THE MECHANISMS OF
DISEASE 3RD EDITION

19. A 44-year-old woman has sought care for the treatment of headaches that have
been increasing in severity and frequency, and has been subsequently diagnosed
with migraines. Which of the following teaching points should her care
provider emphasize?
A) Weight loss and exercise are very important components of your
treatment.
B) Stopping all of your current medications, even temporarily, should provide
some relief.
C) It would be helpful for you to take control of your diet, sleep schedule, and
stress levels.
D) Your headaches are likely a result of nerve disorder and, unfortunately,
cannot be treated successfully.
20. Which of the following principles should underlie the pain control strategy in
the care of a child with a diagnosis of cancer?
A) Opioids should be avoided in order to prevent liver and kidney insult.
B) Dosing and timing should aim for a steady serum level of the drug that is
chosen.
C) Doses of analgesia should be given only when the patients pain becomes
severe.
D) Drugs from numerous classifications should be used to maximize pain
control.

Answer Key

1. A
2. B
3. A
4. A
5. B
6. A
7. D
8. C
APPLIED PATHOPHYSIOLOGY A CONCEPTUAL APPROACH TO THE MECHANISMS OF
DISEASE 3RD EDITION

9. C
10. D
11. C
12. B
13. C
14. D
15. B
16. C
17. A
18. D
19. C
20. B

Chapter 11 Altered Hormonal and Metabolic Regulation

1. A predominant effect of a prolonged excessive growth hormone level is: A)


short stature with obesity.

B) high androgen hormone levels.


C) increased blood glucose levels.
D) insulin-like growth factor (IGF) depletion. 2. The most common cause of
hypothyroidism is:

A) goiter.
B) myxedema.
C) thyroidectomy.
D) autoimmune thyroiditis.
3. Thyroid hormone deficit , which alters the function of all major
organs in the body.
A) decreases metabolism
APPLIED PATHOPHYSIOLOGY A CONCEPTUAL APPROACH TO THE MECHANISMS OF
DISEASE 3RD EDITION

B) increases protein synthesis


C) causes vitamin deficiencies
D) enhances absorption of glucose
4. The most common cause of thyrotoxicosis is Graves disease, which has the
distinguishing characteristic of in addition to a diffuse goiter.
A) muscle fatigue
B) facial myxedema
C) ophthalmopathy
D) decreased cholesterol
5. The major adrenal cortical hormones are steroids and are synthesized from
acetate and:
A) ACTH.
B) albumin.
C) amino acids.
D) cholesterol.
6. Primary adrenal insufficiency is manifested by: A)
truncal obesity and edema.

B) hypokalemia and hypervolemia.


C) hyponatremia and hypoglycemia.
D) hypopigmentation and hypertension.
7. One of the earliest signs of Cushing syndrome is the loss of variable diurnal
secretion of cortisol-releasing hormone (CRH) and:
A) GH.
B) TSH.
C) DHEA. D) ACTH.

8. The iatrogenic form of Cushing syndrome is caused by: A)


long-term cortisone therapy.
APPLIED PATHOPHYSIOLOGY A CONCEPTUAL APPROACH TO THE MECHANISMS OF
DISEASE 3RD EDITION

B) pituitary tumor-secreting ACTH.


C) benign or malignant adrenal tumor.
D) ectopic ACTH secreting lung tumor.
9. The major manifestations of Cushing syndrome include: A)
excessive salt loss.

B) muscle hypertrophy.
C) overt diabetes mellitus.
D) hair and weight loss.
10. The immune suppressive and anti-inflammatory effects of cortisol cause: A)
moderate insulin resistance.

B) increased capillary permeability.


C) increased cell-mediated immunity.
D) inhibition of prostaglandin synthesis.
11. Which of the following individuals is experiencing the effects of a primary
endocrine disorder?
A) A patient with adrenal cortical insufficiency due to pituitary hyposecretion
of ACTH
B) A patient who has hypothyroidism as a result of low TSH production
C) A patient whose dysfunctional hypothalamus has resulted in endocrine
imbalances
D) A patient who has low calcium levels because of the loss of his
parathyroid gland
APPLIED PATHOPHYSIOLOGY A CONCEPTUAL APPROACH TO THE MECHANISMS OF
DISEASE 3RD EDITION

Which of the following


12. physiologic processes is a direct effect of the release of growth hormone by the
anterior pituitary?
A) Development of cartilage and bone
B) Production of insulin-like growth factors (IGFs) by the liver
C) Increase in overall metabolic rate and cardiovascular function
D) Positive feedback of the hypothalamic-pituitary-thyroid feedback system
13. Which of the following individuals displays the precursors to acromegaly?
A) An adult with an excess of growth hormone due to an adenoma
B) A girl who has been diagnosed with precocious puberty
C) An adult who has a diagnosis of Cushing syndrome
D) A patient who has recently developed primary adrenal carcinoma
14. Which of the following residents of a long-term facility is exhibiting signs and
symptoms that are indicative of hypothyroidism?
A) An 80-year-old woman who has uncharacteristically lost her appetite of
late and often complains of feeling cold
B) A 90-year-old woman with a history of atrial fibrillation whose
arrhythmia has recently become more severe
C) An 88-year-old man with a history of Alzheimer disease who has become
increasingly agitated and is wandering more frequently
D) A 91-year-old man with a chronic venous ulcer and a sacral ulcer who has
developed sepsis
15. Abnormal stimulation of the thyroid gland by TSH-receptor antibodies is
implicated in cases of:
A) Cushing syndrome.
B) Graves disease.
C) Addison disease.
D) Cushing disease.
APPLIED PATHOPHYSIOLOGY A CONCEPTUAL APPROACH TO THE MECHANISMS OF
DISEASE 3RD EDITION

Which of the following


16. A patient has developed the facial appearance that is characteristic of
myxedema, along with an enlarged tongue, bradycardia, and voice changes.
treatment modalities is most likely to benefit this
patient?
A) Synthetic preparations of T3 or T4
B) b-Adrenergic blocking drugs and antithyroid drugs
C) Corticosteroid replacement therapy
D) Oral or parenteral cortisol replacement
17. A 33-year-old patient has been admitted to the hospital for the treatment of
Graves disease. Which of the following assessments should the patients care
team prioritize?
A) Assessment of the patients level of consciousness and neurologic status
B) Assessment of the patients peripheral vascular system and assessing for
thromboembolism
C) Assessment of the patients vision and oculomotor function
D) Cardiac monitoring and assessment of peripheral perfusion
18. A patient who has been taking 80 mg of prednisone, a glucocorticoid, each day
has been warned by his primary care provider to carefully follow a plan for the
gradual reduction of the dose rather than stopping the drug suddenly. What is
the rationale for this directive?
A) Sudden changes in glucocorticoid dosing may reverse the therapeutic
effects of the drug.
B) Stopping the drug suddenly may shock the HPA axis into overactivity.
C) Sudden cessation of a glucocorticoid can result in adrenal gland necrosis.
D) Stopping the drug suddenly may cause adrenal insufficiency.
19. The signs and symptoms of abrupt cessation of pharmacologic glucocorticoids
closely resemble those of:
A) Addison disease.
B) Cushing disease.
APPLIED PATHOPHYSIOLOGY A CONCEPTUAL APPROACH TO THE MECHANISMS OF
DISEASE 3RD EDITION

Which of the following


C) Cushing syndrome.
D) Graves disease.
20. pathophysiologic phenomena may result in a diagnosis of Cushing disease?
A) Hypopituitarism
B) Excess ACTH production by a pituitary tumor
C) Autoimmune destruction of the adrenal cortex
D) Malfunction of the HPA system

Answer Key

1. C
2. D
3. A
4. C
5. D
6. C
7. D
8. A
9. C
10. D
11. D
12. B
13. A
14. A
15. B
16. A
17. C
APPLIED PATHOPHYSIOLOGY A CONCEPTUAL APPROACH TO THE MECHANISMS OF
DISEASE 3RD EDITION

Which of the following


18. D
19. A
APPLIED PATHOPHYSIOLOGY A CONCEPTUAL APPROACH TO THE MECHANISMS OF
DISEASE 3RD EDITION

20. B

Chapter 12 Altered Reproductive Function

1. Acute cervicitis is an inflammation of the cervix characterized by:


A) abscess formation.
B) mucopurulent drainage.
C) thick gray-white plaques.
D) persistent pruritic vulvitis.
2. Carcinoma of the cervix is often considered to be a sexually transmitted disease
associated with viral infection.
A) Chlamydia trachomatis
B) herpes simplex
C) human papilloma
D) varicella zoster
3. Pelvic inflammatory disease, an inflammation of the female upper reproductive
tract, is caused by:
A) chronic endometriosis.
B) ruptured tubal pregnancy.
C) STD polymicrobial infection.
D) serous luteal ovarian cysts.
4. Endometriosis is characterized by painful hemorrhagic lesions in the pelvis and
complications that include:
A) pelvic adhesions.
B) endometrial cancer.
C) candidiasis vaginitis.
D) bladder herniation.
5. Leiomyomas are smooth muscle fibroid tumors that usually develop in: A)
fibrocystic breasts.
APPLIED PATHOPHYSIOLOGY A CONCEPTUAL APPROACH TO THE MECHANISMS OF
DISEASE 3RD EDITION

B) postmenopause.
C) uterosacral ligaments.
D) the corpus of the uterus.
6. Symptoms of difficulty in emptying the bladder, frequency, and urgency of
urination are common in women with: A) rectocele.

B) cystocele.
C) endometritis.
D) prolapsed uterus.
7. Although there are no effective screening methods for ovarian cancer, and early
symptoms are usually absent, there are high-risk indicators that include:
A) nulliparity.
B) lactation.
C) mammary duct ectasia.
D) oral contraceptive use.
8. Polycystic ovary syndrome (PCOS) is ovarian dysfunction caused by a
combination of hormone imbalances that include levels.
A) absent FSH
B) insulin deficit
C) elevated LH
D) low androgen
9. In the lactating woman, mastitis is usually the result of: A)
intraductal papillomas.

B) secretory cell hyperplasia.


C) fibrocystic tissue changes.
D) ascending bacterial infection.
10. Women are screened regularly for risk factors and manifestations of breast
cancer. High-risk women may have a history of:
A) late menopause.
APPLIED PATHOPHYSIOLOGY A CONCEPTUAL APPROACH TO THE MECHANISMS OF
DISEASE 3RD EDITION

B) leiomyoma tumors.
C) STDs and vaginitis.
D) multiple pregnancies.
11. Which of the following signs and symptoms is most likely to accompany a
diagnosis of vulvodynia?
A) Vulvar pain
B) Purulent discharge
C) Urinary incontinence
D) Open lesions on the surface of the vulva
12. A 21-year-old college student has sought care because of the vaginal burning,
itching, and redness that have become worse in recent weeks. Which of the
clinicians assessment questions is most likely to apply to a diagnosis of
vaginitis?
A) Have your periods been regular in the last few months?
B) Have you ever had a sexually transmitted infection?
C) Are you using oral contraceptives?
D) Have you ever been pregnant?
13. A 29-year-old patient had a Papanicolaou smear performed during her most
recent visit to her primary care provider. This diagnostic procedure aims to
identify:
A) dysplastic cervical cells.
B) human papillomavirus (HPV) antibodies.
C) cervical polyps.
D) lesions at the transformation zone.
14. Which of the following physiologic processes is caused by estrogens?
A) Increased release of gonadotropin-releasing hormone (GnRH)
B) Stimulation of lactation in the postpartum period
C) Promotion of ovarian follicle growth
D) Progesterone synthesis
APPLIED PATHOPHYSIOLOGY A CONCEPTUAL APPROACH TO THE MECHANISMS OF
DISEASE 3RD EDITION

15. Which of the following disorders of the female genitourinary system is most
likely to result from a disruption in normal endocrine function?
A) Ovarian cancer
B) Pelvic inflammatory disease (PID)
C) Polycystic ovary syndrome (PCOS)
D) Cervicitis
16. Which of the following complaints by middle-aged women should prompt a care
provider to rule out the possibility of ovarian cancer?
A) Im having a lot of vaginal discharge lately and its quite foul.
B) My periods have become quite irregular since last winter.
C) I have a sharp, stabbing pain on my side for the last few days.
D) Im having a lot of indigestion and bloating, which are both new to me.
17. A nurse is conducting a healthy living workshop to a group of younger women.
Which of the following screening recommendations should the nurse provide to
the participants?
A) Monthly breast self-examination (BSE)
B) BSE or mammography each year starting at age 40
C) Annual clinical breast examination until age 65
D) Mammography and clinical breast examination every 3 years until age 40
18. Which of the following processes is a component of the pathogenesis of
proliferative breast lesions without atypia?
A) Growth of ductile or lobular epithelial cells
B) Cystic dilation of terminal ducts
C) Increase in fibrous breast tissue
D) Fat necrosis leading to lesion formation
19. Which of the following physiologic changes results in menopause?
A) Changes in anterior pituitary function that alter ovarian hormone
production
B) Gradual resistance of ovarian target cells to LH and FSH stimulation
APPLIED PATHOPHYSIOLOGY A CONCEPTUAL APPROACH TO THE MECHANISMS OF
DISEASE 3RD EDITION

C) Cessation of ovarian function and decreased estrogen levels


D) Decreased levels of gonadotropin-releasing hormone (GnRH)
20. A 59-year-old woman with a recent diagnosis of breast cancer has begun a course
of hormone therapy. What is the goal of this pharmacologic treatment?
A) Blocking the effects of progesterone on tumor growth
B) Increasing serum hormone levels to promote tumor cell lysis
C) Blocking the entry of malignant cells into the axillary lymph nodes
D) Blocking receptors on the surface of malignant cells

21. Smooth muscle relaxation and shunting of blood into the sinusoids is mediated
by and results in a penile erection.
A) norepinephrine
B) nitroglycerine
C) nitric oxide
D) nicotinic acid
22. Priapism is a condition that causes ischemia as a result of:
A) circumcision trauma.
B) failure of detumescence.
C) tight retracted foreskin.
D) fibrous plaque in the penis.
23. Common risk factors for erectile dysfunction due to generalized penile arterial
insufficiency include:
A) cryptorchidism.
B) cigarette smoking.
C) testicular torsion.
D) benign prostate hypertrophy.
24. Squamous cell cancer of the penis is more likely to develop in men with chronic:
A) erectile dysfunction.
APPLIED PATHOPHYSIOLOGY A CONCEPTUAL APPROACH TO THE MECHANISMS OF
DISEASE 3RD EDITION

B) herpes ulcerations.
C) Peyronie disease.
D) smegma accumulation.
25. In a hydrocele, excess fluid is present in the:
A) epididymis.
B) tunica vaginalis.
C) pampiniform plexus.
D) vas deferens ampulla.
26. Testicular torsion, a serious disorder affecting young male individuals, causes: A)
inguinal herniation.

B) cancer of the scrotum.


C) dartos muscle atrophy.
D) loss of testicular perfusion.
27. Men older than age 50 are at high risk for prostatic hypertrophy with
complications that include:
A) hypospadias.
B) scrotal edema.
C) urine retention.
D) testicular cancer.
28. The major cause of acute prostatitis is: A)
prostate hyperplasia.

B) acute pyelonephritis.
C) gram negative
D) mucous gland overgrowth.
E) coli.
29. The most important factor in the evaluation and treatment of benign prostatic
hypertrophy (BPH) is considered to be: A) frequency of erectile dysfunction.
APPLIED PATHOPHYSIOLOGY A CONCEPTUAL APPROACH TO THE MECHANISMS OF
DISEASE 3RD EDITION

B) testosterone level management.


C) prostate cancer prevention measures.
D) subjective symptoms reported by the patient.
30. Cryptorchidism, or undescended testes, is a direct cause of: A)
infertility.

B) paraphimosis.
C) prostate cancer.
D) low testosterone.
31. Which of the following physiologic processes results from the synthesis and
release of testosterone?
A) Protein catabolism
B) Musculoskeletal growth
C) Release of luteinizing hormone (LH) and follicle-stimulating hormone
(FSH)
D) Prostatic hyperplasia
32. A 41-year-old patient has undergone a vasectomy. What is the physiologic basis
for this contraception technique?
A) Spermatogenesis is inhibited because sex hormones may no longer
stimulate the Sertoli cells.
B) Spermatozoa can no longer reach the epididymis and do not survive.
C) The rete testis becomes inhospitable to sperm.
D) Sperm can no longer pass through the ductus deferens.
33. A patient has been diagnosed with an anterior pituitary tumor, and synthesis and
release of follicle-stimulating hormone has become deranged. What are the
potential consequences of this alteration in endocrine function?
A) Dysfunction of spermatogenesis
B) Overproduction of luteinizing hormone
C) Inhibition of testosterone synthesis
D) Impaired detumescence
APPLIED PATHOPHYSIOLOGY A CONCEPTUAL APPROACH TO THE MECHANISMS OF
DISEASE 3RD EDITION

34. Which of the following factors constitutes the most significant risk for balanitis
xerotica obliterans?
A) Multiple sexual partners
B) Androgen deficiency
C) Uncircumcised penis
D) Chronic prostatitis
35. Which of the following disorders of the male genitourinary system creates the
most urgent need for prompt and aggressive medical treatment?
A) Spermatocele
B) Benign prostatic hyperplasia (BPH)
C) Intravaginal testicular torsion
D) Erectile dysfunction
36. A 30-year-old man has been diagnosed with mumps orchitis, a disease that has
the potential to result in:
A) hematuria.
B) hematocele.
C) sterility.
D) penile atrophy.
37. After seeking care due to recent history of testicular enlargement and scrotal pain,
a 22-year-old college student has been diagnosed with testicular cancer. Which
of the patients following statements indicates the need for further teaching?
A) I cant shake this feeling like Ive received a death sentence.
B) I have to admit that the prospect of losing a testicle is a bit overwhelming.
C) I really hope the cancer hasnt spread anywhere, because Ive read that its a
possibility.
D) I guess theres some solace in the fact that this cancer wasnt a result of an
unhealthy lifestyle.
38. Which of the following assessments is most likely to reveal a potential
exacerbation in a 70-year-old patients diagnosis of benign prostatic hyperplasia
(BPH)?
APPLIED PATHOPHYSIOLOGY A CONCEPTUAL APPROACH TO THE MECHANISMS OF
DISEASE 3RD EDITION

A) Urine testing for microalbuminuria


B) Blood test for white blood cells and differential
C) Bladder ultrasound
D) Sperm morphology testing
39. Which of the following statements about screening for prostate cancer is most
accurate?
A) Digital rectal examination detects the majority of new cases of prostate
cancer.
B) A positive prostate-specific antigen (PSA) test is definitive for prostate
cancer.
C) BPH and prostatitis can confound prostate screening results.
D) Digital rectal examination and PSA testing have been proven ineffective.
40. Which of the following diagnoses is most likely to require surgical correction?
A) Hypospadias
B) Orchitis
C) Erectile dysfunction
D) Spermatocele

Answer Key
1. B
2. C
3. C
APPLIED PATHOPHYSIOLOGY A CONCEPTUAL APPROACH TO THE MECHANISMS OF
DISEASE 3RD EDITION
APPLIED PATHOPHYSIOLOGY A CONCEPTUAL APPROACH TO THE MECHANISMS OF
DISEASE 3RD EDITION

34. C
35. C
36. C
37. A
38. C
39. C
40. A

Chapter 13 Altered Ventilation and Diffusion

1. Pleuritic chest pain associated with respiratory movements is usually


described as:
A) bilateral.
B) localized.
C) continuou
s.
D) substernal
.
2. A man sustained a puncture injury to his chest that caused a tension
pneumothorax to form. This is a life-threatening condition because: A)
expired air exits the bleeding wound.

B) trapped, inspired air collapses the lung.


C) the opposite lung hyperinflates.
D) blebs on the lung surface rupture.
3. Atelectasis is most commonly caused by: A)
airway obstruction.

B) pulmonary embolism.
C) inflammation of the pleura.
D) chronic forceful coughing.
4. Acute onset bronchial asthma causes wheezing and breathlessness as a result of:
APPLIED PATHOPHYSIOLOGY A CONCEPTUAL APPROACH TO THE MECHANISMS OF
DISEASE 3RD EDITION

A) airway inflammation.
B) alveolar collapse.
C) compression atelectasis.
D) pulmonary hypertension.
5. Emphysema is characterized by hyperinflation of the lungs that produces
increased:
A) total lung capacity.
B) alveolar wall thickness.
C) a1-antitrypsin enzyme.
D) submucosal gland hypertrophy.
6. As a result of hypoxemia and polycythemia, persons with chronic obstructive
bronchitis are prone to:
A) breakdown of elastin.
B) left-sided heart failure.
C) pulmonary hypertension.
D) expiratory airway collapse.
7. With bronchiectasis, persistent airway obstruction and chronic infection results in
bronchial:
A) dilation.
B) rupture.
C) infarction.
D) thickening.
8. Cystic fibrosis (CF), the major cause of severe chronic respiratory disease in
children, is characterized by:
A) constipation.
B) salt depletion.
C) watery mucus.
D) high cholesterol.
9. The adverse effects of emboli on the pulmonary circulation and airways include:
APPLIED PATHOPHYSIOLOGY A CONCEPTUAL APPROACH TO THE MECHANISMS OF
DISEASE 3RD EDITION

A) excess surfactant.
B) area bronchodilation.
C) local vasoconstriction.
D) lower lobe consolidation.
10. The common results of respiratory failure are hypoxemia and:
A) hypercapnia.
B) bradycardia.
C) vasoconstriction.
D) pulmonary emboli.
11. A patient with a history of emphysema is experiencing hypoxemia after a taxing
physical therapy appointment. Which of the following physiologic phenomena
will occur as a consequence of hypoxemia?
A) Peripheral vasodilation
B) Necrosis
C) Hypoventilation
D) Increased heart rate
12. An elderly patient who has been restricted to bed by numerous comorbidities for
several weeks has been diagnosed with a large pleural effusion. Which of the
following treatment modalities is most likely to resolve the patients most recent
health problem?
A) Thoracentesis
B) Supplementary oxygen therapy
C) Administration of corticosteroids
D) Administration of bronchodilators
13. A motor vehicle accident has resulted in a tension pneumothorax for the driver of
the car. Which of the following manifestations is associated with tension
pneumothorax?
A) Audible friction rub over the affected lung
B) Mediastinal shift
C) Metabolic alkalosis
APPLIED PATHOPHYSIOLOGY A CONCEPTUAL APPROACH TO THE MECHANISMS OF
DISEASE 3RD EDITION

D) Atrial fibrillation
14. A patient has just been admitted to the postsurgical unit following a below-
theknee amputation. Which of the following measures should her care team
prioritize to prevent atelectasis during the patients immediate recovery?
A) Bedrest and supplementary oxygen by nasal cannula
B) Administration of bronchodilators by nebulizer
C) Deep-breathing exercises and early mobilization
D) Adequate hydration and a high-humidity environment
15. Which of the following manifestations typically accompanies an asthmatic
attack?
A) Decreased residual volume
B) Decreased pulmonary arterial pressure
C) Prolonged inspiration
D) Hyperinflation of the lungs
16. A 51-year-old man has been diagnosed with chronic bronchitis after a long
history of recurrent coughing. Which of the mans following statements
demonstrates a sound understanding of his new diagnosis?
A) If I had quit smoking earlier than I did, I think I could have avoided getting
bronchitis.
B) Im pretty sure that I first caught bronchitis from the person who has the
cubicle next to mine at work.
C) I read on the Internet that I might have got bronchitis because I was born
with an enzyme deficiency.
D) I think that I probably could have prevented this if I had got in the habit of
exercising more when I was younger.
17. Which of the following individuals is experiencing a disorder of ventilation that
has a restrictive rather than obstructive etiology?
A) A 30-year-old African American man who has been diagnosed with
sarcoidosis
B) An infant whose routine screening is suggestive of cystic fibrosis
APPLIED PATHOPHYSIOLOGY A CONCEPTUAL APPROACH TO THE MECHANISMS OF
DISEASE 3RD EDITION

C) An elderly, lifelong smoker who has been admitted to hospital with


emphysema
D) A 16-year-old girl who must limit her activity to prevent asthmatic attacks
18. Prolonged immobility is implicated in the development of which of the following
disorders?
A) Bronchitis and bronchiectasis
B) Sarcoidosis and idiopathic pulmonary fibrosis
C) Atelectasis and pulmonary embolism
D) Pulmonary hypertension and cor pulmonale
19. Which of the following is most likely to precipitate an asthmatic attack in a child
with a diagnosis of extrinsic, or atopic, asthma?
A) Pet dander
B) Cold weather
C) Stress
D) Respiratory tract infections
20. Pain is an expected assessment finding in patients who have which of the
following lung diseases?
A) Asthma
B) Pleural effusion
C) Pulmonary arterial hypertension (PAH)
D) Pleuritis

Answer Key

1. B
2. B
3. A
4. A
5. A
6. C
APPLIED PATHOPHYSIOLOGY A CONCEPTUAL APPROACH TO THE MECHANISMS OF
DISEASE 3RD EDITION

7. A
8. B
9. C
10. A
11. D
12. A
13. B
14. C
15. D
16. A
17. A
18. C
19. A
20. D

Chapter 14 Altered Perfusion

1. The pathophysiology of heart failure involves an interaction between decreased


pumping ability and the to maintain cardiac output.
A) aortic hypertrophy
B) compensatory mechanisms
C) electrical conductivity
D) parasympathetic system
2. Cardiac output is the each minute.
A) volume load
B) blood pumped
C) stroke volume
D) force generated
APPLIED PATHOPHYSIOLOGY A CONCEPTUAL APPROACH TO THE MECHANISMS OF
DISEASE 3RD EDITION

3. One of the principal mechanisms by which the heart compensates for increased
workload is:
A) myocardial hypertrophy.
B) sodium and water retention.
C) endothelin vasoconstrictors.
D) ventricular wall tension increase.
4. In right-sided heart failure, peripheral edema is evidenced by:
A) weight gain.
B) copious urination.
C) shortness of breath.
D) decreased blood pressure.
5. The most common causes of left-sided heart failure include: A)
acute myocardial infarction.

B) chronic pulmonary disease.


C) impaired renal blood flow.
D) tricuspid valve regurgitation.
6. Hypovolemic shock occurs as a result of: A)
myocardial infarction.

B) excessive vasoconstriction.
C) chronic intracellular fluid shift.
D) acute intravascular volume loss.
7. In shock, one of the best indicators of blood flow to vital organs is:
A) warm legs.
B) urine output.
C) blood pressure.
D) consciousness.
APPLIED PATHOPHYSIOLOGY A CONCEPTUAL APPROACH TO THE MECHANISMS OF
DISEASE 3RD EDITION

8. Anaphylactic shock is directly associated with: A)


loss of blood volume.

B) bacterial blood infection.


C) failure of the heart as a pump.
D) type I hypersensitivity response.
9. Severe shock can be followed by acute lung injury/acute respiratory distress
syndrome (ALI/ARDS) characterized by: A) hyperventilation.

B) excessive surfactant.
C) hyperinflated alveolar sacs.
D) ventilation-perfusion mismatch.
10. A common symptom of the ischemia associated with gastrointestinal
redistribution of blood flow is:
A) gastric bleeding.
B) nausea and vomiting.
C) irritable bowel syndrome.
D) copious high volume diarrhea.
11. A patient has been experiencing increasing fatigue in recent months, a trend that
has prompted an echocardiogram. Results of this diagnostic test suggest that the
patients end-diastolic volume is insufficient. Which of the following parameters
of cardiac performance will directly decrease as a result of this?
A) Inotropy
B) Cardiac contractility
C) Preload
D) Afterload
12. Which of the following health problems is associated with heart failure as a result
of diastolic dysfunction?
A) Uncontrolled hypertension
B) Chronic bradycardia
C) Ischemic heart disease
APPLIED PATHOPHYSIOLOGY A CONCEPTUAL APPROACH TO THE MECHANISMS OF
DISEASE 3RD EDITION

D) Myocardial hypertrophy
13. Assessment of an elderly female patient reveals the presence of bilateral pitting
edema of the patients feet and ankles and pedal pulses that are difficult to
palpate. Auscultation of the patients lungs reveals clear air entry to bases, and
the patients oxygen saturation level is 93% and vital signs are within reference
ranges. What is this patients most likely health problem?
A) Right-sided heart failure
B) Left-sided heart failure
C) Cardiogenic shock
D) Cor pulmonale
14. The most recent blood work of a patient with a diagnosis of heart failure indicates
increased levels of atrial natriuretic peptide (ANP) and brain natriuretic peptide
(BNP). What are the most likely effects of these peptides on the patients
physiology?
A) Water retention
B) Increased tubular sodium reabsorption
C) Inhibition of the renin-angiotensin-aldosterone system
D) Sympathetic nervous stimulation
15. A nurse is performing patient health education with a 68-year-old man who has
recently been diagnosed with heart failure. Which of the following statements
demonstrates an accurate understanding of his new diagnosis?
A) Ill be sure to take my beta blocker whenever I feel short of breath.
B) Im going to avoid as much physical activity as I can so that I preserve my
strength.
C) I know its healthy to drink a lot of water, and Im going to make sure I do
this from now on.
D) Im trying to think of ways that I can cut down the amount of salt that I
usually eat.
16. Electrical burns over a large surface area of a patients body have resulted in
hypovolemic shock after the loss of large amounts of blood and plasma. Which
of the following mechanisms is the patients body likely to implement to
compensate for this loss of fluid?
APPLIED PATHOPHYSIOLOGY A CONCEPTUAL APPROACH TO THE MECHANISMS OF
DISEASE 3RD EDITION

A) Increased heart rate


B) Vasodilation
C) Diuresis
D) Inhibition of ADH
17. A patient who developed a deep vein thrombosis during a prolonged period of
bedrest has deteriorated as the clot has dislodged and resulted in a pulmonary
embolism. Which of the following types of shock is this patient at risk of
experiencing?
A) Cardiogenic shock
B) Hypovolemic shock
C) Obstructive shock
D) Distributive shock
18. For which of the following types of shock might intravenous antibiotic therapy be
indicated?
A) Obstructive shock
B) Distributive shock
C) Cardiogenic shock
D) Hypovolemic shock
19. An 86-year-old male patient is disappointed to learn that he has class II heart
failure despite a lifelong commitment to exercise and healthy eating. Which of
the following age-related changes predisposes older adults to developing heart
failure?
A) Increased vascular stiffness
B) Orthostatic hypotension
C) Increased cardiac contractility
D) Loss of action potential
20. A patient with a diagnosis of heart failure has returned from a visit with his
primary care provider with a prescription for a change in his daily medication
regimen. Which of the following drugs is likely to improve the patients cardiac
function by increasing the force and strength of ventricular contractions?
A) A b-adrenergic blocker
APPLIED PATHOPHYSIOLOGY A CONCEPTUAL APPROACH TO THE MECHANISMS OF
DISEASE 3RD EDITION

B) A diuretic
C) A cardiac glycoside
D) An ACE inhibitor

Answer Key

1. B
2. B
3. A
4. A
5. A
6. D
7. B
8. D
9. D
10. A
11. C
12. D
13. A
14. C
15. D
16. A
17. C
18. B
19. A
20. C

Chapter 15 Altered Nutrition


APPLIED PATHOPHYSIOLOGY A CONCEPTUAL APPROACH TO THE MECHANISMS OF
DISEASE 3RD EDITION

1. The adipocytes in adipose tissue not only serve as a storage sites, they also: A)
produce linoleic fatty acid.

B) synthesize triglycerides.
C) increase glucagon release.
D) degrade fat-soluble vitamins.
2. Protein contains nitrogen. A negative nitrogen balance represents: A)
more protein consumed than excreted.

B) a reduced need for nitrogen as protein.


C) more nitrogen excreted than consumed.
D) less use of nitrogen for protein synthesis.
3. Natural appetite suppression mechanisms, necessary for food intake control,
include :
A) ketoacid deficiency.
B) cholecystokinin storage.
C) decreased blood glucose.
D) leptin receptor stimulation.
4. Body weight should be used in combination with other measurements to
establish if a person is underweight or overweight. Obesity is indicated by: A)
female body fat of 20% and 30%.

B) body mass index (BMI) of 30 to 40.


C) relative body weight of 70% to 100%.
D) abdominal fat/ hip ratio of 0.8 to 1.0.
5. A patient with upper body obesity also has central fat distribution. This body fat
configuration places the patient at greater risk for than a patient
with lower body obesity.
A) osteoporosis
B) renal disease
C) cardiometabolic disorders
APPLIED PATHOPHYSIOLOGY A CONCEPTUAL APPROACH TO THE MECHANISMS OF
DISEASE 3RD EDITION

D) chronic anemia
6. As the problem of childhood and adolescent obesity increases, an increase in the
incidence of is occurring in this obese population.
A) type 2 diabetes mellitus
B) attention deficit disorder
C) juvenile rheumatoid arthritis
D) antibiotic-resistant bacterial infections
7. A diet deficient in calories and protein causes marasmus, which is characterized
by:
A) discolored
hair.
B) bradycardia.
C) enlarged
liver.
D) pitting
edema.
8. Protein-calorie malnutrition with loss of lean tissues and muscle mass results in:
A) respiratory muscle stimulation.
B) excessive blood cell production.
C) diarrhea.
D) increased cardiac contractility.
9. Similarities between girls or women with anorexia nervosa and bulimia nervosa
include:
A) periodontal disease.
B) low estrogen level.
C) electrolyte imbalances. D) enlarged parotid gland.

10. Both binge-eating and bulimia nervosa patients consume excessive amounts of
foods secretively. A major difference is that binge-eaters:
A) remain overweight.
B) eat when not hungry.
APPLIED PATHOPHYSIOLOGY A CONCEPTUAL APPROACH TO THE MECHANISMS OF
DISEASE 3RD EDITION

C) are substance abusers.


D) experience depression.
11. A large, high-calorie meal has resulted in the intake of far more energy than a
person requires. What will the individuals body do with the excess
carbohydrates provided by this meal?
A) Convert them into glucose and store them in the liver and muscles
B) Excrete most of the excess polysaccharides through the kidneys
C) Convert the carbohydrates into amino acids in preparation for long-term
storage
D) Create structural proteins from some of the carbohydrates and store the
remainder as triglycerides
12. Which of the following patients is most likely to be in positive nitrogen balance?
A) A patient who is receiving treatment for sepsis
B) A patient whose diagnosis of pneumonia is causing a fever
C) A woman who has been admitted to the hospital in early labor
D) A patient who sustained extensive burns in a recent industrial accident
13. In addition to facilitating bowel movements, a diet that is high in fiber confers
which of the following benefits?
A) Lowering cholesterol and blood glucose
B) Removing toxins and metabolic byproducts
C) Lowering blood pressure and resting heart rate
D) Increasing intestinal absorption of vitamins and minerals
14. Which of the following statements best conveys the endocrine function of
adipose tissue?
A) Adipose tissue antagonizes the effects of insulin on cell membranes.
B) Adipose tissue produces ghrelin, which stimulates both appetite and eating.
C) Adipose tissue produces and secretes cholecystokinin (CCK), which
stimulates the hypothalamic feeding center.
D) Adipose tissue produces leptin, which mediates body weight.
APPLIED PATHOPHYSIOLOGY A CONCEPTUAL APPROACH TO THE MECHANISMS OF
DISEASE 3RD EDITION

15. Chronic inflammation as a result of excess adipose tissue is implicated in the


etiology of which of the following health problems?
A) Osteoporosis
B) Type 2 diabetes
C) Rheumatoid arthritis
D) Systemic lupus erythematosus (SLE)
16. A public health nurse has noted a significant increase in the number of
schoolaged children who are obese. Which of the following factors is most
significant predictor of childhood obesity?
A) Low socioeconomic status
B) Low self-esteem
C) Having obese parents
D) Living in a rural or inner-city neighborhood
17. Which of the following measures should a school nurse prioritize in the treatment
and prevention of childhood obesity?
A) Group cognitive therapy
B) Use of selective serotonin-reuptake inhibitors (SSRIs)
C) Education on exercise and nutrition
D) High-protein, low-carbohydrate diet
18. Which of the following characteristics distinguishes kwashiorkor from
marasmus?
A) Impairment of immune function
B) Lack of dietary fat intake
C) High intake of carbohydrates
D) Impaired pigment synthesis
19. A frail, 87-year-old female patient has been admitted to a hospital after a fall and
has been diagnosed with failure to thrive. Which of the following laboratory
values would suggest that the patient may be experiencing malnutrition?
A) Low prealbumin
B) High C-reactive protein
APPLIED PATHOPHYSIOLOGY A CONCEPTUAL APPROACH TO THE MECHANISMS OF
DISEASE 3RD EDITION

C) High bilirubin
D) Low fasting blood sugar
20. Which of the following assessments should be prioritized in the care of a patient
with anorexia nervosa?
A) Serum electrolyte levels
B) Chest auscultation
C) White blood cell count with differential
D) Blood pressure monitoring

Answer Key

1. B
2. C
3. D
4. B
5. C
6. A
7. B
8. C
9. C
10. A
11. A
12. C
13. A
14. D
15. B
16. C
17. C
18. C
APPLIED PATHOPHYSIOLOGY A CONCEPTUAL APPROACH TO THE MECHANISMS OF
DISEASE 3RD EDITION

19. A
20. D

Chapter 16 Altered Elimination

1. A major complication of persistent gastroesophageal reflux is:


A) strictures.
B) heartburn.
C) chest pain.
D) hoarseness.
2. Acute gastritis refers to a transient inflammation of the gastric mucosa that is
most commonly associated with:
A) diarrhea.
B) food allergies.
C) gastric reflux.
D) alcohol intake.
3. The Helicobacter pylori protobacteria cause peptic ulceration by producing:
A) acids
B) toxins
C) ischemia
D) bleeding
4. A hallmark of irritable bowel syndrome is abdominal pain: A)
relieved by defecation.

B) most severe at night.


C) with blood in the stool.
D) after and between meals.
5. Inflammatory bowel diseases are accompanied by systemic manifestations that
include:
A) autoimmune anemia.
APPLIED PATHOPHYSIOLOGY A CONCEPTUAL APPROACH TO THE MECHANISMS OF
DISEASE 3RD EDITION

B) rheumatoid arthritis.
C) thrombocytopenia.
D) lactose intolerance.
6. Crohns type of inflammatory bowel disease is characterized by: A)
granulomatous lesions.

B) ulcerative erosions.
C) fibrotic smooth muscle.
D) necrotic crypt abscesses.
7. Unlike the Crohn type of inflammatory bowel disease, the ulcerative colitis type
is characterized by:
A) skip lesions.
B) steatorrhea.
C) gastric ulcers.
D) pseudopolyps.
8. Diverticulitis, a complication of diverticulosis, is manifested by acute:
A) rectal bleeding.
B) abdominal distention.
C) large-volume diarrhea.
D) lower left quadrant pain.
9. Major causes of mechanical bowel obstruction include: A)
chemical irritation.

B) ruptured appendix.
C) abdominal distention.
D) postoperative adhesions.
10. As a protective measure to keep abdominal inflammation and infection localized,
the peritoneum:
A) constricts bowel contents.
APPLIED PATHOPHYSIOLOGY A CONCEPTUAL APPROACH TO THE MECHANISMS OF
DISEASE 3RD EDITION

B) secretes fibrous exudate.


C) increases intestinal motility. D) abdominal vasoconstriction.

11. Which of the following signs and symptoms most clearly suggests the need for
endoscopy to rule out esophageal cancer?
A) Heartburn after an individual consumes high-fat meals
B) Dysphagia in an individual with no history of neurologic disease
C) A new onset of gastroesophageal reflux in a previously healthy individual
D) Recurrent episodes of gastritis that do not respond to changes in diet
12. A 60-year-old male patient has presented to his primary care provider to follow
up with his ongoing treatment for peptic ulcer disease. What is the most likely
goal of this patients pharmacologic treatment?
A) Inhibiting gastric acid production
B) Promoting hypertrophy of the gastric mucosa
C) Increasing the rate of gastric emptying
D) Increasing muscle tone of the cardiac sphincter
13. Which of the following individuals most likely faces the greatest risk of
developing Clostridium difficile colitis?
A) A 55-year-old man who takes proton pump inhibitors for the treatment of
peptic ulcers
B) A 79-year-old hospital patient who is being treated with broad-spectrum
antibiotics
C) A premature neonate who has developed hyperbilirubinemia and is
receiving phototherapy
D) A 30-year-old patient who has a history of Crohn disease and has been
admitted to a hospital to treat a recent flare-up
14. A 66-year-old woman has been diagnosed with diverticular disease based on her
recent complaints and the results of a computed tomography (CT) scan. Which
of the patients following statements demonstrates an accurate understanding of
this diagnosis?
A) From now on, Im going to stick to an organic diet and start taking more
supplements.
APPLIED PATHOPHYSIOLOGY A CONCEPTUAL APPROACH TO THE MECHANISMS OF
DISEASE 3RD EDITION

B) I think this might have happened because Ive used enemas and laxatives
too much.
C) Ive always struggled with heartburn and indigestion, and I guess I shouldnt
have ignored those warning signs.
D) I suppose I should try to eat more fiber and become a bit more active.
15. An ultrasound has confirmed appendicitis as the cause of a 20-year-old mans
sudden abdominal pain. Which of the following etiologic processes is implicated
in the development of appendicitis?
A) Obstruction of the intestinal lumen
B) Elimination of normal intestinal flora
C) Sloughing of the intestinal mucosa
D) Increased osmolality of intestinal contents
16. Which of the following characteristics differentiates inflammatory diarrhea from
the noninflammatory type?
A) Larger volume of diarrhea
B) Electrolyte imbalances
C) Absence of blood in the stool
D) Infection of intestinal cells
17. Which of the following patients should the nurse observe most closely for the
signs and symptoms of paralytic ileus?
A) A patient who is postoperative day 1 following gall bladder surgery
B) A patient whose acute diarrhea has necessitated the use of antidiarrheal
medications
C) An obese patient who refuses to ambulate because he complains of
shortness of breath
D) A patient with a longstanding diagnosis of irritable bowel syndrome
18. Which of the following meals is most likely to exacerbate an individuals celiac
disease?
A) Spaghetti with meatballs and garlic bread
B) Stir-fried chicken and vegetables with rice
APPLIED PATHOPHYSIOLOGY A CONCEPTUAL APPROACH TO THE MECHANISMS OF
DISEASE 3RD EDITION

C) Oatmeal with milk, brown sugar, and walnuts


D) Barbecued steak and a baked potato with sour cream
19. Which of the following statements is true of colorectal cancer? A)
Aspirin and NSAIDs are implicated in the etiology.

B) It is one of the few cancers that is known to sometimes have an infectious


etiology.
C) Most cases are quite advanced before symptoms become apparent.
D) Survival rates for colorectal cancer are less than 20%, but are increasing.
20. Following the analysis of a recent barium enema and colonoscopy with biopsy, a
patient has been diagnosed with colorectal cancer. Which of the following
treatment modalities will be the mainstay of this patients treatment?
A) Chemotherapy
B) Radiation therapy
C) Pharmacologic therapies
D) Surgery

Answer Key
1. A
2. D
3. B
4. A
5. A
6. A
7. D
8. D
9. D
10. B
11. B
APPLIED PATHOPHYSIOLOGY A CONCEPTUAL APPROACH TO THE MECHANISMS OF
DISEASE 3RD EDITION

12. A
13. B
14. D
15. A
16. D
17. A
18. A
19. C
20. D

Chapter 17 Degenerative Changes in Aging

1. The nurse keeps the environment warmer for older adults because they are more
sensitive to cold because of the age-related changes in their:

a. metabolism rate.
b. subcutaneous tissue.
c. musculoskeletal system.
d. peripheral vascular system.

ANS: B

The reduction of subcutaneous tissue as an age-related change causes sensitivity to


cold because it is the main insulator of the body.

2. The nurse reassures the distressed 75-year-old male that the wartlike dark macules
with distinct borders are not melanomas, but the skin lesions of:

a. senile lentigo.
b. cutaneous papillomas.
c. seborrheic keratoses.
d. xerosis.

ANS: C
APPLIED PATHOPHYSIOLOGY A CONCEPTUAL APPROACH TO THE MECHANISMS OF
DISEASE 3RD EDITION

Dark, slightly raised macules are seborrheic keratoses, which may be mistaken for
melanomas.

3. The nurse is accompanying a group of older adults on a July 4th outing to monitor
heat prostration. Older adults are intolerant of heat because of an age-related
reduction of:

a. melanin.
b. perspiration.
c. body temperature.
d. capillary fragility.

ANS: B

Reduction in perspiration related to reduced sweat gland function results in possible


heat intolerance from an inability to cool the body by evaporation.

4. The nurse cautions the CNAs to use care when transferring or handling older adults
because their vascular fragility will cause:
a. altered blood pressure.
b. pressure ulcers.
c. pruritus.
d. senile purpura.

ANS: D

Increased capillary fragility results in subcutaneous hemorrhage or senile purpura


from incautious handling by caregivers.

5. The nurse assesses a stage I pressure ulcer on an older adults coccyx by the
appearance of a:

a. clear blister.
b. nonblanchable area of erythema.
c. scaly abraded area.
d. painful reddened area.

ANS: B
APPLIED PATHOPHYSIOLOGY A CONCEPTUAL APPROACH TO THE MECHANISMS OF
DISEASE 3RD EDITION

A red nonblanchable area is indicative of a stage I pressure ulcer.

6. The CNA caring for an older adult asks if the yellow, waxy, crusty lesions on the
patients axilla and groin are contagious. The nurses most helpful response is:

a. Yes. It is cellulitis caused by bacteria.


b. No. It is seborrheic dermatitis caused by excessive sebum.
c. Yes. It is an indication of scabies.
d. No. It is the lesion seen with basal cell carcinoma.

ANS: B

Seborrheic dermatitis is a bothersome skin condition resulting from an excess of


sebum.
7. The nurse leads a group of postmenopausal older women on a daily 15-minute
walking tour through the long-term care facility to:

a. improve bone strength.


b. orient them to their surroundings.
c. improve their socialization.
d. increase their appetite.

ANS: A

Stress to long bones by weight-bearing and walking will increase bone strength.

8. When the perplexed 70-year-old woman asks, How in the world can my bones be
brittle when I eat all the right foods? the nurses most informative reply is:

a. Calcium loss is expected in the older adult.


b. Calcium is continuously withdrawn from bone for nerve and muscle function.
c. Smoking and alcohol consumption speed calcium loss from the bones.
d. Walking and standing increase calcium loss from the bone.

ANS: B
APPLIED PATHOPHYSIOLOGY A CONCEPTUAL APPROACH TO THE MECHANISMS OF
DISEASE 3RD EDITION

Calcium is constantly withdrawn from the bone for nerve and muscle function and
clotting needs.

9. When the 70-year-old woman complains, I weigh exactly the same as I did when I
wore a size 10 and now I can barely squeeze into a size 16, the nurse explains:

a. Metabolism in the older adult creates increased adipose tissue.


b. Postmenopausal women gain adipose tissue related to loss of calcium.
c. Decrease in muscle mass is replaced with adipose tissue.
d. Kyphosis causes a redistribution of weight.

ANS: C
Decrease in muscle mass is replaced with adipose tissue, which frequently changes
the appearance of the body, but not the weight.

10. When the 70-year-old postmenopausal woman asks whether her hormone
replacement therapy (HRT) will prevent bone loss, the nurses most helpful response
is:

a. No. HRT is not helpful after the age of 60.


b. Yes. HRT will prevent bone loss but can cause a stroke, heart attack, or breast
cancer.
c. No. HRT is reliant on some natural estrogen production from the ovaries.
d. Yes. HRT is a widely accepted therapy for prevention of bone loss.

ANS: B

HRT is helpful to prevent bone loss, but the risks of cardiovascular complications and
cancer have made the choice of HRT controversial.

11. An 80-year-old-woman who has osteoarthritis complains of how ugly her hands
have become since she has developed Heberden nodes, which are:

a. yellow longitudinal lines in the nails.


b. thickened discolored fingernails.
c. darkened areas under the fingernail.
d. bony enlargements of distal joints of the fingers.
APPLIED PATHOPHYSIOLOGY A CONCEPTUAL APPROACH TO THE MECHANISMS OF
DISEASE 3RD EDITION

ANS: D

Heberden nodes are bony enlargements of the distal joints of the fingers associated
with osteoarthritis.

12. The nurse modifies the nursing care plan for a 62-year-old woman in an
extendedcare facility who is suffering a flare in her rheumatoid arthritis to include
interventions to:

a. increase fluid intake.


b. schedule several rest periods to balance activity.
c. reduce salt in the diet.
d. assist with rigorous finger extension exercises.

ANS: B

Balancing rest and activity allows the resident to remain relatively flexible. Joints may
be splinted to reduce contracture.

13. The nurse explains that emphysema is a chronic obstructive pulmonary disease
characterized by the pathophysiology of:

a. constriction of the bronchial tree, excessive mucus, and nonproductive cough.


b. calcification of the alveoli and a dry cough.
c. overinflation of the alveoli, making them ineffective for gas exchange.
d. inflammation of the trachea and bronchioles, excessive mucus, and
productive cough.

ANS: C

Emphysema causes overinflation of the nonelastic alveoli, which disallows gas


exchange in the affected alveoli and results in reduced oxygenation.

14. The nurse explains that the pathophysiology of a myocardial infarct is that:

a. a portion of the myocardium necroses and scars over.


b. the coronary vessels are narrowed during the attack.
APPLIED PATHOPHYSIOLOGY A CONCEPTUAL APPROACH TO THE MECHANISMS OF
DISEASE 3RD EDITION

c. the ischemic myocardium causes pain during the attack but is able to
regenerate.
d. there is damage to the myocardium but no serious alteration of cardiac output.

ANS: A

The myocardium necroses and scars and does not regenerate. The degree of heart
damage is related to the amount of necrosis.
15. The nurse is aware that the cardinal signs and symptoms of congestive heart
failure are:

a. dyspnea and edema.


b. myocardial pain and hypotension.
c. ventricular arrhythmias and cyanosis.
d. atrial arrhythmias and polycythemia.

ANS: A

Dyspnea and generalized edema are the cardinal signs and symptoms of congestive
heart failure.

16. The nurse explains that pernicious anemia is caused by:

a. an iron deficiency.
b. a deficiency of vitamin B12.
c. inadequate nutrition.
d. blood loss.

ANS: B

Pernicious anemia results from a deficiency of vitamin B12.

17. The nurse alters the nursing care plan for a patient with a hiatal hernia and
resultant gastrointestinal reflux to include interventions for:

a. encouraging the patient to lie down after meals.


b. drinking two full glasses of liquid after the evening meal.
APPLIED PATHOPHYSIOLOGY A CONCEPTUAL APPROACH TO THE MECHANISMS OF
DISEASE 3RD EDITION

c. eating smaller, more frequent meals.


d. using caffeine drinks to assist with digestion.

ANS: C

Eating smaller and more frequent meals does not enlarge the stomach.

18. The nurse suspects that the pale, edematous, listless diabetic patient who has a
blood urea nitrogen (BUN) level of 35 mg/dL and a creatinine level of 4 mg/dL has:
a. diverticulitis.

b. congestive heart failure.


c. chronic renal failure.
d. benign prostatic hypertrophy.

ANS: C

The increased BUN and creatinine levels indicate renal failure.

19. The most appropriate intervention added to the nursing care plan for a person with
Parkinson disease with a nursing diagnosis of Nutrition, less than body requirements
related to difficulty swallowing, would be to:

a. feed the patient at each meal.


b. place the patient in a semi-Fowler position for mealtime.
c. offer a thick, high-nutrition shake as a snack.
d. encourage the patient to drink a sip of water after each bite of solid food. ANS:

Thick shakes are easier to swallow without aspiration and will also improve nutrition.

20. The nurse would anticipate that a person with a hemorrhagic CVA to the left
hemisphere would exhibit:
APPLIED PATHOPHYSIOLOGY A CONCEPTUAL APPROACH TO THE MECHANISMS OF
DISEASE 3RD EDITION

a. language disturbances.
b. poor impulse control.
c. inappropriate affect.
d. confabulation.

ANS: A
A left hemisphere CVA would most likely cause language disturbances such as
aphasia, agraphia, or alexia.

Chapter 18 Integrated Pathophysiologic Concepts: Diabetes Mellitus

1. Type 1 diabetes mellitus results from destruction of the pancreatic beta cells by
two mechanisms. The mechanism for type 1A diabetes is destruction.
A) genetic
B) resistant
C) idiopathic
D) autoimmune
2. The metabolic abnormalities that lead to type 2 diabetes include: A)
chronic overeating.

B) insulin resistance.
C) acute pancreatitis.
D) recurrent hypoglycemia.
3. Which test provides a way to monitor fluctuations of blood glucose levels over
the previous 6 to 12 weeks?
A) Glucose tolerance test
B) Fasting blood glucose
C) Capillary blood glucose
D) Glycosylated hemoglobin
4. Diabetic ketoacidosis (DKA) more commonly occurs in patients with type 1
diabetes, when the lack of insulin leads to the mobilization of that causes
excess ketone production by the liver.
APPLIED PATHOPHYSIOLOGY A CONCEPTUAL APPROACH TO THE MECHANISMS OF
DISEASE 3RD EDITION

A) cortisol
B) fatty acids
C) potassium
D) bicarbonate
5. Factors that contribute to the severe hyperglycemia that precipitates
hyperglycemic hyperosmolar state (HHS) include: A) fluid
retention and edema.

B) thromboembolism formation.
C) insulin overdose.
D) glycosuria and water loss.
6. A change in the circadian rhythm for glucose tolerance and an inappropriate
increase in counterregulatory hormones can lead to in diabetics.
A) hypoglycemia
B) Somogyi effect
C) hyperinsulinemia
D) dawn phenomenon
7. In people with type 1 diabetes, the beneficial effects of exercise also carry an
increased risk of:
A) rapid weight loss.
B) respiratory disorders.
C) rebound hyperglycemia.
D) profound hypoglycemia.
8. One of the first renal manifestations of diabetic nephropathy is:
A) microalbuminuria.
B) oliguria.
C) hypertension.
D) hyperlipidemia.
APPLIED PATHOPHYSIOLOGY A CONCEPTUAL APPROACH TO THE MECHANISMS OF
DISEASE 3RD EDITION

9. Diabetic retinopathy, the leading cause of acquired blindness in the United States,
is characterized by retinal:
A) glaucoma.
B) hemorrhages.
C) dehydration.
D) infections.
10. Impaired and delayed healing in a person with diabetes is caused by chronic
complications that include:
A) ketoacidosis.
B) Somogyi effect.
C) fluid imbalances.
D) chronic neuropathies.
11. A hospital patient with a diagnosis of type 1 diabetes has been administered a
scheduled dose of regular insulin. Which of the following effects will result
from the action of insulin?
A) Promotion of fat breakdown
B) Promotion of glucose uptake by target cells
C) Promotion of gluconeogenesis and protein synthesis
D) Initiation of glycogenolysis
12. A patient with longstanding type 2 diabetes is surprised at his high blood sugar
readings while recovering from an emergency surgery. Which of the following
factors may have contributed to the patients inordinately elevated blood glucose
levels?
A) The tissue trauma of surgery resulted in gluconeogenesis.
B) Illness inhibited the release and uptake of glucagon.
C) The stress of the event caused the release of cortisol.
D) Sleep disruption in the hospital precipitated the dawn effect.
13. The results of a 44-year-old obese mans recent diagnostic workup have
culminated in a new diagnosis of type 2 diabetes. Which of the following
pathophysiologic processes underlies the patients new diagnosis?
APPLIED PATHOPHYSIOLOGY A CONCEPTUAL APPROACH TO THE MECHANISMS OF
DISEASE 3RD EDITION

A) Beta-cell exhaustion due to long-standing insulin resistance


B) Destruction of beta cells that is not attributable to autoimmunity
C) T-lymphocytemediated hypersensitivity reactions
D) Actions of insulin autoantibodies (IAAs) and islet cell autoantibodies
(ICAs)
14. Which of the following assessment findings of a male patient constitutes a
criterion for a diagnosis of metabolic syndrome?
A) The patient states that he does less than 30 minutes of strenuous physical
activity each week.
B) The patients resting heart rate is typically 85 to 95 beats per minute.
C) The patients blood pressure is consistently in the range of 140/90 mm Hg.
D) The patient has a family history of type 2 diabetes.
15. Which of the following pregnant women likely faces the greatest risk of
developing gestational diabetes?
A) A patient who was diagnosed with placenta previa early in her pregnancy.
B) A patient who is gravida five (in her fifth pregnancy)
C) A patient who has hypertension and elevated triglycerides
D) A patient who is morbidly obese
16. A patients primary care provider has ordered an oral glucose tolerance test
(OGTT) as a screening measure for diabetes. Which of the following
instructions should the patient be given?
A) The lab tech will give you a sugar solution and then measure your blood
sugar levels at specified intervals.
B) Youll have to refrain from eating after midnight and then go to the lab to
have your blood taken first thing in the morning.
C) Theyll take a blood sample and see how much sugar is attached to your red
blood cells.
D) You can go to the lab at any time; just tell the technician when you last ate
before they draw a blood sample.
17. Which of the following insulin-administration regimens is most likely to result in
stable blood glucose levels for a patient with a diagnosis of type 1 diabetes?
APPLIED PATHOPHYSIOLOGY A CONCEPTUAL APPROACH TO THE MECHANISMS OF
DISEASE 3RD EDITION

A) One large dose of long-acting insulin each day before breakfast


B) Intermediate-acting insulin at 8:00 AM and 8:00 PM with rapid-acting
insulin before each meal
C) Six to eight small doses of rapid-acting insulin each day, with capillary
monitoring before each
D) Long-acting insulin twice daily (breakfast and bedtime), with
intermediateacting insulin in the afternoon
18. A hospital patient has been complaining of increasing fatigue for several hours
and his nurse has entered his room to find him unarousable. The nurse
immediately checked the patients blood glucose level, which is 22 mg/dL (1.2
mmol/L). The nurse should prepare to administer which of the following?
A) A snack that combines simple sugars, protein, and complex carbohydrates
B) A 50% glucose solution intravenously
C) Infusion of rapid-acting insulin
D) Oral solution containing glucagon and simple sugars
19. A diabetic patients most recent blood work indicated a decreased glomerular
filtration rate and urine testing revealed microalbuminuria. Which of the
following self-care measures should the patients care team suggest to the
patient?
A) Use of over-the-counter diuretics
B) Increased fluid intake
C) Decreased oral sugar intake
D) Measures to lower blood pressure
20. Which of the following comorbidities represents the greatest risk for the
development of foot ulcers in a diabetic patient?
A) Distal symmetric neuropathy
B) Previous incidents of diabetic ketoacidosis
C) Diabetic nephropathy
D) Autonomic neuropathy
APPLIED PATHOPHYSIOLOGY A CONCEPTUAL APPROACH TO THE MECHANISMS OF
DISEASE 3RD EDITION

Answer Key

1. D
2. B
3. D
4. B
5. D
6. D
7. D
8. A 9. B

10. D
11. B
12. C
13. A
14. C
15. D
16. A
17. B
18. B
19. D
20. A

You might also like